Download as pdf
Download as pdf
You are on page 1of 135
MODULAR SYSTEM AREAS of Polygonal Regions Ahmet Gakir Zambak Copyright © Sirat Baa Reklamalk ve Etim Aragon San, Tic. AS. All rights reserved No part ofthis book may be reproduced, stored ina retrieval system or hansmitted in any form without the prior writen permission ofthe publisher Digitol Assembly ombok Typesetting & Design Page Design Samil Keskinoglu Language Proofreader oe Barnett Publisher Slat Basim Reklamk ve Etim Aral Son Ti. AS Printed by (CaBlayan AS. SarnicYolu Uzeri No:7 Gas tami, September 2008 ‘Tel: +90-0-282-252.22 +90-0-232-522-20-96-97 ISBN: 978-975-266930-7 Printed in Turkey DISTRIBUTION, ZANBAN YAYINLARI Bulgurlu Mah, Hominne Cesmesl Sok No, 20 34696 Uskudar / Istanbul Tel: +90-216 522 09 00 (pbx) Fax: +90-716 443 98:39 hitp://book zambak com PREFACE i aa To the Teacher This book is a high school course about the areas of polygonal and circular regions. Before studying this book, students should have a basic understanding of the properties of triangles, polygons and ctrces, although you may choose to teach this book in stages, studying first triangles and then the areas of triangles, followed by quadritaterals and then the areas of quadrilaterats, etc. This decision will depend on your teaching schedule and curriculum. The book is divided into three sections. Section 1 begins with basic definitions related to area, and then leads students through a study of the areas of triangles Each important theorem, formula or property ts treated as a subtoptc, and self-check questions follow these subtopics. In many cases the book shows tmportant conclusions. Once students can understand and apply these conclusions, they will be able t0 solve more problems than they can solve using only the formulas and properties on their own. © Section 2 looks at the areas of quadrilaterals A number of properties of area are common to all quadrilaterals, so these are given first. For similar reasons, parallelograms are introduced next, since many quadrilaterals are also parallelograms. Once students have understood these first two parts of the section, they should be able to understand the remaining parts (about rhombt, squares, trapezoids and kites) easily. Section 3 covers the areas of regular polygons, circles, and parts of circles. Students learn about mscribed and circumscribed circles and polygons, and the relation of the tnradius and circumradius to the area ofa polygon This ts followed by a study of the areas of sectors, segments and rings The final part of the section tooks at the properties of similarity in circles: This book follows a step-by-step teaching approach which leads the student from basic definitions and concepts t0 a gradual mastery of the topic through a large number of clear, sotved examples. At each stage, students can check their understanding by trying problems in self-check Check Yourself sections. There are also many further exercises atthe | | PES SOS end of each section, and a Chapter Summary and Chapter Review Tests are given at the end of the book. Each Chapter Review Test ts based on a different section of the book, and provides practice for exams. To the Student This hook is about the areas of polygonal regions and circles. We use area very often in our daily lives, for example to Jind ont how many tiles we need 10 nile a floor In many ways studying this topic 1s like studying a purale: if you enjoy puzzles then you will enjoy this part of geometry, To succeed at it, just learning formulas is not enough: yon also have to make decisions and use your powers of logic. Although this is difficult at first, if you try hard to solve the questions then you will begin to enjoy geometry Acknowledgensents Many friends and colleagues helped and supported me during the writing of this book, and T would hike ro thank att af them for their comments and suggestions, in writing and in conversations. In particular I would like to thank — Mutammer Taskavan and Mustafa Kirtkst for thetr advice and very valuable encouragement. 1 am also grateful to Samil Keskinoglu for his patient typesetting and expert design, and to Zoe Barnett for her careful proofreading Finally, Txcould lke to thank my family for their patience while Iwas working on this book Ahmet Qaler — ww we ee ee This book has been designed so that you can use it effectively Each section has its own special color that you can see at the bottom of the page ee cae ieee ene ete ‘When te ao 2 cues aa ee etna fe teaear iene and _-| twice! Make sure you understand it Definition boxes ive formal descriptions of TEREST new concepts. Theorem boxes contain las Pears oe eS propositions that can be proved. Rule and Postulate boxes contain tmportant rules that we use to solve questions. The information in these bores ts very important for further The stance ene the center understanding and for solving problems. The aca ofa wang shal ofthe po The area ofa retangle ithe prover f Inia sansa Examples show you problems and their solution, with explanations EXIGE 1 two sides of 1 aa nitrate eater Gok tonfeadthom ab bathe bo What isthe as Solution Lette sigs Questions in Check Yourself sections help you | Check Yourself 1 check your understanding of what you have Just studied. Solve these questions alone and then compare your answers wath the answer key provided. If your answers are correct, you can move on the next section. If an answer is | 2 tn the fgue, ABCD 4 ecangle with AD = 8 em wrong, go through your working again and | and m( 248) = 30" Find acancn) check back through the examples in the section. de, Find the area of this retanle reminds you of material that ts related to the topic you are studing. Notebook text helps you to remember the ‘math you need so that you can understand the A small notebook tn the left or right margin of a page material. It might help you to see your mistakes, too! Exercises at the end of each section cover the ‘matertal in the whole section. You should be able ? tosalveall the problems which do not hare a star. | A. One star (@) next to a question means the 1 Arosa her question is a bit more difficult. Some of these ‘umes 30 on. questions are similar to the questions in the International Mathematical Olympiads, but are a bit easier. The answers to the exercises are at the back of the book 10.4 Concept Check question e and m(ZBEC) = 20° Given that AE = 2and EC = 6, find book sept of Area ite ee San Se The Chapter Summary summarizes all the S| eeee st, ‘The centroid ofa angle divides each median in] the mato 12 ‘ material that has been covered in the chapter The Concept 1. Concept of Area Check section contains oral questions. You do not need paper +The union of pogon and is nero mon is ald a OF Bem 10 answer these questions. If you answer Concept Check questions correctly, it means you know the topic! The answers to these questions are in the material you studied Go back over the material if you are not sure about an answer Finally, Chapter Review Tests contain multiple-choice questions in increasing order of difficulty to help you prepare for exams. Test I 1s about the first section of the book, Test 2 is related to the second section aud Test & is about the third section. The answer key for these tests is at the bach of the 1. AREAS f Polygonal Regions AREAS OF TRIANGULAR REGIONS..10 ‘A. THE CONCEPT OF AREA. 1. Basic Definitions, 2. Area of a Rectangle B. AREA OF A TRIANGLE Theorem: area of a right triangle... Theorem: area of an equilateral triangle . Theorem: Heron’s Formula ‘Theorem: trigonometric formula for the area of a triangle Theorem: area of a triangle by its Inradlus,....24 Theorem: area of a triangle by its clreumradius coe C. PROPERTIES OF THE AREA OF A TRIANGLE Pick’: Theorem EXERCISES 1 “4 Ae AREAS OF QUADRILATERALS... A. AREA OF A QUADRILATERAL Theorem: area of a convex polygon B. AREA OF A PARALLELOGRAM.. 52 Theorem: area of a parallogram..... C. AREA OF A RHOMBUS.. 'D, AREA OF A SQUARE. E. AREA OF A TRAPEZOID... Theorem: area of a trapezoid «..... F. AREA OF AKITE Prowng the Pythagorean Theorem 81 EXERCISES 2... 83 3. AREAS OF REGULAR POLYGONS AND CIRCLES... A. AREA OF A REGULAR POLYGON ‘Theorem: area of a regular polygon. 8. AREA OF A CIRCULAR REGION... ‘Theorem: area of a circle... C. AREA OF A SECTO! ‘Theorem: area of a sector 1. D. AREA OF A SEGMEN’ ‘Theorem: area of a segment, E. AREA OF A RING. F. RATIOS IN CIRCLE: Three Unsolved Problems 110, EXERCISES : CHAPTER SUMMARY..... CONCEPT CHECK....... CHAPTER REVIEW TEST 4 sa-csesseeeeseenve ANSWERS Yor cleressrrercrseeseerrs TST GLOSSARY esscsssceseesseseesssnseeeernaneseees TB INTRODUCTION ‘There are many branches of geometry, including Euclidean geometry, analytic geometry projective geometry and combinatorial geometry. This book is about area, which is one of the ‘miost Important topics in Euclidean geometry, Here are some real-world problems that we can solve with a knowledge of area: + Ifyou are buying or rentng a house, how much will you pay? The price you pay will depend partly on the area of the house, + Imagine you want to make some clothes from a piece of material. If you know the area of the material, you will knew how many items of clothing you can make from tt. + Imagme you are at a pizza restaurant that sells small and medium pizzas with diameters 12 cm and 24 cm, Which is bigger: two small pizzas or one medium pizza? Asyou can see, knowing about the areas of shapes such as quadrilaterals and exeles ean help us m our daly lives. Geometers throughout story haye found formulas to calculate the areas of figures. But where did it all begin? Why did people need geometry? ‘Thousands of years ago, geometry helped to solve practical problems and make life easter for people. In Egypt, geometry was very important. The river Nile flooded nearly every vear, and people used geometry to find the correct location and boundaries of thetr lands after a flood. ‘Very old documents from around 3100 BC show that people m Egypt and Mesopotamia used mathematical rules and techniques to find land areas, calculate volumes and construct buildings. In one later Egyptian document, the writer Ahmes wrote the area of an asosceles beh triangle with base b and height has A Chmese mathematicians were using the Pythagorean Theorem (without its proof) in around 1100 BC, and Hindu mathematicians also used many rules based on the geometzie properties of figures. Starting in the 6th century BC, the ancient Greeks took these geometric rules from the Egyptians, Babylonians and others and generalized them. They gave Uns new knowledge the name ‘geomet’, which comes ftom the Greek words geo (meaning Earth), and metron (measure). The Greeks began studving geometry in an organized way; Thales and his follower Pythagoras wrote Logical proofs of formulas, and used geometry as a branch of science. The famous philosopher Plato also insisted that everybody had to learn geometry before they began studying at his school. = In the 3rd century BC the Greek mathematician Euclid wrore a famous collection of books ‘about geometry called Elements, In these books. Euctid proved the geometrical formulas of ‘his tme by starting from just a few sumple statements. He also described how geometry could be used in astronomy and map-making. After the Greeks, the Romans used geometry effectively for city planning, measurmg land and building weapons, although they did not use proofs. Muslim mathematicians also began to study geometry, They translated many Greek books, includmg Euctid’s Elements, mto Arabic, With these translations, Mustin mathematicians developed geometry in many ways: ‘AL-Khwarizmi calculated as 3.1416, and Al-Kashi calculated the first fourteen digits of correctly. Ibn al-Havtham studied non-Fuclidean geometry and described the bases of amalytie geometry. Al-Quhi was one of the greatest geometers. New European ‘mathematicians also used the Arabic translations of some books when the originals were lost. In the 17h century the French mathematician Descartes desenbed analytic geometry formally, Fermat also studied analytic geometty but he didi publish his work, and so many ‘people say that Descartes 1s the father of analytic geometry. Analytic geometry 3s the study of algebraic solutions to geometric problems, and.it makes it easier to solve some problems. For instance, we can fd the area of a polygon usmg algebra if we use coordmates to desenbe the location of its vertices. After the discovery of analytic and projective geometries, geometers in the 18th and 19th centunes developed conics and the analyneal analysis of ‘ines and circles, ‘Today we can say that geometry is all around us. It is used in particular in algebra and trigonometry, engineering, architecture, art, industrial design, computing and graphic design. Computer-based simulation isa new and important area of study that uses geometry: “If we have a good knowledge of geometry, many things become simpler: geometry 15. Anowledge, but it 1s also funn, And so let us begin our study of this important tople. tegior AREAS OF TRIANGULAR REGIONS A. THE CONCEPT OF AREA 1. Basic Definitions polygonal region. ‘The umon of a polygon and its mterior regon 1s called a polygonal region We name polygons by their vertices. For example, AABC 18 the name of a mangle with ks sane [vertices at pomts A, B and C, and ABCD 4s the name ofa quadnilateral with vertices at pomts S mangle 4, B, Cand D. We use extra notation to refer F 4 quadristers 5 pentagon, | 8 polygonal region: (AABC) 4s a triangular F @ —bexagon region, and (ABCDE) 1s a pentagonal region > % epson in the figure, (ABCDEF) 3s the umion of the & fase Thexagon and its interior region. Since : 10 |ABCDEF 1s a hexagon, we can say that ; (ABCDEF) is a hexagonal region ¢ square wut The intenor region of a square with side length one um 1s called a square unit. We wnte uumit® to mean a square unit. In the figure opposite, each side of the square measures 1 unit and so its area 1s 1 square ‘um, or 1 umit®, We can also use metric umts Jorlengths and azeas: a square with side lem hhas area 1 cm’, and a square with side 1m shas area 1m’, ete. The area of a closed plane figure is the (otal number of non-overlapping square units and part ‘units that cover the surface of the polygonal region. The area of a figure 1s always a positive real number, We use the letter A to mean the area of a polygon: the area of ABC is A(AABC), and the area of the pentagon ABCDE 1s A(ABCDE). If the sides ofa figure are not natural numbers or if the polygon 1s very big, 11s dificult 10 find its area by counting the mdividual umt squares. In tis book we will learn a set of formulas and methods to find the area of any geometric figure by calculation. Aves of Pelygonal esions alutude, herght ‘An alutude is a Line segment between a vertex of a polygon and a Line contamung a side of the polygon, which as perpendicular to thas line. The length of an altirude is called a height of the polygon. We wnite ly ly ete. to mean the altitudes to sides a, b, ete, of a polygon. ase ‘The side ofa polygon from which we draw an altitude is called the base. We can use any side of a polygon as a base. alscude tase alaiade ‘ase Tse eetangle triangle parallelogr: Note Im an isosceles triangle, the congruent sides ae called the legs of the mangle and the third side 1s the base. area congruence postulate If two figures are congruent then their areas are the same. ‘Two polygons congruent if comesponding sides and] anges are the same 2. Area of a Rectangle The area of a rectangle 1s the product of the lengths of two consecutive sides: A(ABCD) = ab a [REUSE 10 stces ofa rectangle measure 14 em and 20 em = What is the area of this rectangle? Solution Let the sides of the rectangle be a = 14 cmand D = 20cm, Then A=a-b= 14-20 = 280 cm’ Arras of Pangular Regions == 2 Solution Solution = 4 Arrectangle has area 84 cm and one of its sides measures 7 cm, What is this perimeter of this rectangle? Let us wnite a= 7emand A = 84 em*. SoA = ab gives us 84 =7-b,ie.b = 12 em. So the perimeter of the rectangle is 2(a +b) = 2(7 + 12) = 38 em. In the figure, ABCD is a rectangle wath a : AC = 10 cmand AB = 8 cm, Find the area of this rectangle. We can use the Pythagorean Theorem or special right mands to find the length of BC = b, By the Pythagorean Theorem, AB? + BC’ = AC* 840" = 10° b =6em. So A(ABCD) = ab = 6-8 = 48 cm’. Arectangee has area 108 em! and one of ts sides measures three umes the other side, Find the penmeter of this rectangle Let a = x and b = 3x be the side lengths since one side is three times as long as the other side. Then A = ab gives us 108 = x - 3x = 3x", so Soa = Gcmandb = 18 em. So the perimeter of ABCD is 2 (a + b) = 2- G + 18) If we connect a pomt on one side of a rectangle to the endpoints of the opposite side then the area of the triangle obtained is, half the area of the rectangle: in the figure, Avoa of Polygon Reson = 5 Solution Arras of Pangular Regions In the figure, ABCD is a rectangle with sides AB = 8 cmand AD = 6 cm, and E isa pont on side DC. Find the combined area of the shaded regons. ss uN —z By the pevous mule, A(aABE) = ACABCD) So the sum ofthe shaded areas wil ato be ABCD) So the combined area is MABED) _6°8 94 em? Check Yourself 1 1. Axectangle has permeter 40 cm and one side 1s 4 cm longer than the other side. Find the area of this zectangle. 2 In the figure, ABCD 1s a rectangle with BD = 8 em D ie and m(ZABD) = 30°. Fmd A(ABCD), I] a @ 3. A rectangle has area 48 cm" and perimeter 28 cm. Find the lengths of the sides of this rectangle, 4. One side of a rectangle 1s twice as long as another side. Given that the perimeter of this rectangle is 30 om, find its area. Answers 2 163 em* 3. Gem, 8em 4. 50 cm’ B. AREA OF A TRIANGLE base-height formula The area of a triangle is half the product of the length of one base and the height of the triangle from that base: m the figure, h A(AABC) = [Look at the figure 1m AABC, BC = a We draw a hne parallel to BC Unrough A, and Sheets erick A's{ ftom Band C we draw perpendiculars BB! ates ti SH) and CC’ to the paralle hne ed ace opposite C1 We can say that BCC’B’, BHAB’ and AHCC’ cc = aH Jn a wang, the side are rectangles, and also BB’ Also, AABH = ABAB’ since ZABH = 2™ Tom. So A(AABC) In the given figure, ABCD is a rectangle with m(ZABD) = 30°, m(ZBEC) = AF = 12and AE = 8. Find the area of quadrilateral ABC. ADP 1s 2307-60" 90" tangle so AD= 1 Since ABCD sa rectangle, BC = AD = 6. AEBC ss an sosceles night tiangle (because m(BEC) = m(ZECB) = 43°), so BB = BC = 6. So in wetangle ABCD, AD = a = Gand AB =b = AE + EB =8+6=14. 6-68, 6-6 2 3 Finally, A@ECF) = A(ABCD) ~ (A(AADF) + A(AEBC)) = 6-14-( = 84— (183 + 18) = 66 - 18V3. Avoa of Polygon Reson Proof Es 14 Solu ES 15 Solu EE 16 Solution Arras of Pangular Regions area-of- an-qqunlaterai-trangle ‘The area of an equilateral mangle with side length a is one-fourth of the product of a” and J/3: im the figure, A(AABC) ote Let us draw the altitude AH m AABC. A } since AABC 6 equilateral, (ze) = 60° ana so att = n, = 95. . 50 A(aABC) a Pe (One side of an equilateral tangle measures 6 cm. Find the area of this tangle. By the theorem above, A WB _ 6 7 Sree en The height of an equilateral triangle 1s 10 cm, Find its area. avs 20 ste neighe« n= 2° 10, 20 0 = 2% em ight + ¥ 20 BB _ 100.6 _ 1006 .,, a a So A(AABC) The area and perimeter of an equilateral tangle have the same value. Find the length of one side of this tangle, we Let the side measure a, then the area is “Y° and the perimeter is 34 os 3a.and 0 ANS 1s the length of one side of the wiangle. Ir the area and the perimeter have equal values then Check Yourself 3 1. The hypotenuse of a nght tangle measures 25 units and one of its legs measures 24 units, Find the area of this wangle. An isosceles right triangle has a hypotenuse of 10 units. Find its area In the figure, ABCD 3s a rectangle, c EB = 8, m(ZBEC) = 48° and N BC = DC. Find hearea otacs, | \, a 2 Find the area of the equilateral triangle with the given side length. ad » 10 5. Find the area of the equilateral triangle with the given height us D8 2B Answers 18 2.29 3.3N9 4aa3 bas 5.0 1965 p Sas eas and rearrange the parts to get the second triangle When we do this, we can see that there is an empty space in the second ~~~} triangle, Where is the missing square? If we draw the figures accurately and with a large scale, we will see that the slopes of the red and blue tangles are different, This means that the bigger shapes are not triangles. So we cannot calculate the areas directly by using the area formula, and in fact the areas are not equal. Proof HERON OF ALEXANDRIA (10-75 AD) asa of lain ve Gs, thin wd erate ve Tired in Aland in Expt. ewe was nese in the pac sy mecha sd eninering. le so stdid amt, epi, ananow ad citer Ta gone, be fad he seo ah by igen adhe eh he aden He ered ny Arras of Pangular Regions Heron's Formule: Ifa ange has sides a, b and cand perimeter " 2u then the area of the triangle is the square root of the product of a, w - a, u—b and uc: im the figure, ya atbte a if z A(AABC) = Ju anu =F) =e) Look at the figure. Let us draw the alutude ¥ AH to BC and let AH = h, BH HC = a—x and A(ABC) = A 3 By the Pythagorean Theorem in AABH and AAHC respectively we get Mai=e, and (a —x)" +h From (1) and (2) we have i? ao sata "(ax)", 2) * ~@-x of = (a? av + (expand the bmomial) fayfs ota? + 2ax 3" (sumplity) +e Le. x 2ax (3) rearrange) Let us use (3) in (1): h? = e* — (ac)? ~(a* ae So 4a°’ = Qac ~(a" H+) - Qac +a°—H +c), (A) (ifference of two squares) Equalizmg denommators gives us fr ‘We know that A(AABC) a ° 16A" = (6*— (a? — 2ae + C))- (@ + 2ac + €*—b?) (from (4)) 168A" = 6 @ 0) (a + 0)" BY) (use a’ + 2c + =a +0) IGA" = a t+ yb +a—Cla + e-ba te +b). (S) (difference of two squares) ‘We know that the perimeter of ABC is 2u.ie.a +b +¢=2u Cac ett in: 18a = Ox 2024 22H Dhan, LAF = BAAD 242-9 and so A= qit-@—a)-(u—b)-(@ =o), as required. 17 Find tne area ofthe tnange with side lengths 4 em, 5 em and 7 em. atbte 44547 2 2 ACABC)= E-DE-BVS—D = fF 1S 1=4YT em* We can use Heron’s Formula with 8 [ERIE 18 the aces ofa triangle ABC ate a = 7, b = Sand ¢ = 12, Find h, 749412 2 A(AABC)= fl U4 = 7) (4 = 8) (4 = 12) = 14.5. and 3 By Herons's Formula with = 14 we have 12-h, Also, A(AABC) = "6. 0 145° migonomemne formula for-the grea oF a mangle ‘The area of a triangle is half the product of any two sides and the sine of the angle between these two sides: m the figure, Proof Lookat the figure. Let us draw the altitude from the vertex A to side BC, so AH = h,, From the figure, Se acaasc)= "7 dab smc. The proofs for the other paits of sides are similar, Avoa of Polygon Reson Em 19 Solution Ens 21 Solution Arras of Pangular Regions In the figure, AB BC = a = 6 emand m(ZB) = 60° What is A(AABC)? 4 3em, 4 We know that a = 6, ¢= 3 and m(ZB) = 60". since acsey= a-snB mae oS, A(aaBC) = 2.6-3.sin60° = 25 em: 2 2 In the figure, AD = 9 om, BD = Gem, AE = 8 cmand EC = 20m, What is AGADE) , ‘ACQABC) ‘We can fd both of the areas using the trigonometne formulas we have just seen, We know AD = 9, AE = 8, AB = 11 and AE = 10, so 1 AG@ADE) _3°AD- AB sin A ‘A(QABC) AD-AE_ 9-8 4 ABAC 110 11 2. AB. AC. sin A a In the figure, AB BC = 8 em, AC = 3V2 cm and m(ZC) = 43° Find &. We have AB = ¢ = 6, BC = a = Sand AC 3, and we know sin 45° ee Since ACAABC) = Fab sin = jac sin a, we have 34-42 =6.sino. which gives us smo 4, 16, = 36° oro = 190° Since m(Z0) = 45%, 0 = 90 is impossible, and so a = 30°. QQ the ses of aPOR are p = 3,q = 12 andr = 15. Find sm R. Solution Let us find A@PQR) by using both Heron's Formula and the trigonometne formula for the area of a triangle. B412415 By Heron's Formula yath w= ""—"—"=16, A(APOR)= Jf U0 = 5)- U6 = 12) G6 =15) = 81 1 zed sink= > 9:12. sm =30. sink. S030. sm R= 6/TT, and sosin n= SME _ AMET By the trigonomettic formula, A(APQR. Check Yourself 4 1. Atange has sides of 13 cm, 14cm and 15 em. Find its area. 2 The sides of a tangle are @ = 3, b Find the three heights of this tangle. 3. AB = 8 cmand AC = 6V3 cm are two sides of a triangle ABC. Find A(AABC) if Sande a. m(ZA) = 30", b. m(ZA) = 60° ce. m(ZA) = 90" 4. The sides of a triangle are a = 8, b = 7 and c = 9, What is the sine of angle B? Answers iH, 2 as 3. 12V3 om" 36 em’ 2408 emé 4. 1, 84m". h, 4a 3 morrele; fnscribed gircle, incenter_tnradius ‘The meucle (or inscnbed cucle) ofa tangle isa circle inside the triangle which is tangent to cach of ts sides. The center of the mcircle is called the incenter. It Les at the intersection point of the angle bisectors of the triangle. The radius of the incircle is called the inradius (r) area af @ transle be its inraduis ‘The area of a triangle 1s the product of half ats perimeter and its inradius: in the figure, fa +b +¢=2u then JN AQABC) = 0+ © [A A a Avoa of Polygon Reson Proof EES 23 Solution Ex 24 Solution Arras of Pangular Regions Look at the figure. Let J be the mcenter of r AABC and let r be its inradius. If we connect rand the vernices A,B and C we ean wnte A(AABC) = A(AAIB) + A(ABIC) + A(AAIC) fp ow Lo™ i “ A triangle with perimeter 20 cm has inradius 6 em. Find the area of this triangle: Ir the perimeter 1s 2u then 2u = 20, Le." = 10. By the theorem we have just seen, A= "+r = 10-6 = 60 em" The sides of a tangle measure 7, 8 and 9 units, Find the radius of the incircle of this mangle +84 2 w= =12 A= fi aa Da = BIZ HUZ—HAZ—9)—(leron’s Formula ) = VP T3-125 Also, A= u-r = 12-1 = 1245, sor = V5, circumcircle, circumscribed eixcle, eircumradius. ‘The eiele which passes through all three vertices of a triangle is called the circumeizcle (or circumsenbed circle) of the mange. The radius of the circumencle 1s called the crcumradius (R), Proof Solution qrea-of strane befis-greumradias ‘The area of a wiangle 1s equal to the ratio of the product of the sides to four nmes its curcumradus: an the igure, @ bie The law of smes tells us that 2 = - © — ap, Soa =2R-sin Ate. sina=—. aR ‘Substituting this in A(AABC) = .b-c-sinA gives us A(AABC) = 1.b.c.2 = 276 as = 2 # 27 OR aR” required. The sides of a triangle measure 8 em, 10 cm and 12 cm. Find the circumradius R of this mangle Let w see 19, then by Heron's Formula the area of the tnangle will be A= (B- 8-1) -1)= YR S= Tem * abe off = a ves us 157 = 8-10-12 167 Using the formula A(AABC) = Te te R= Som ‘The sides of a right triangle measure 6 cm and & cm. Find the sum of the circumradius and the mradius of this tangle. From geometry we inow that if the vertices of a right triangle all le on the same circle then the hypotenuse ts the diameter of this circle, So the hypotenuse 1s the diameter of the carcumeucle (2R). By the Pythagorean Theorem, 6° + §° = (2R)*. So 2R = 10,1e. R= Sem. Also, ithe mangle aright tangle then ts areas A="-£ = 98 94 mt atbte_6+8+10 2 So the sum of the eircumradius and mnradus is R + r= 9 +2 = Tom. Let n= =12, then sce A = ar we have 24 = 12-1, 16. r= 2em, Avoa of Polygon Reson ES 27 Solution Acircle has radius 8 cm. Find the area of the equilateral triangle whose vertices lie on this, curcle. ‘We can wnite the area of the equilateral triangle in two ways: «i aaa 42238 ana a= a 4k anda = 83, 4 48 Check Yourself 5 1. The legs of a right triangle measure 5 cm and 12 em. Find the mradius of this tangle. 2. I ABC, m(ZA) = 90° and AB = 3¥9. If m(2C) = 45°, fmd the circumradius of AABC. ‘The sides of a tangle measure 12, 9 and 7 ums, Fmd the lengths and r of the cireumrodsus and ‘amediur of this tianle 4 One ofthe legs ofa night tangle measures 9 unis and the diameter of ts eicumseirle ‘5 15, Find the radius of ths angle oO. An equilateral triangle has side length 6. Find the sum of the mradius and cireumradius of ts trang 6. The base of an sosols triangle mensures 24 units andthe other sides are euch 13 uns Jong Find the sradius rand exeumradiusR of ts tangle Answers ram 23 aR a3 oa C. PROPERTIES OF THE AREA OF A TRIANGLE Arras of Pangular Regions So far we have learned several formulas for the area of a triangle. However, sometimes these formulas may not be enough to solve a problem, or using them can make the solution longer. In this section we look at some properties of triangles that can help us to solve problems more atrectly Proof EX 28 Solution — Proof Em 29 In a nght wiangle, the product of the Legs 1s, 7 equal (0 the product of the hypotenuse and the length of the altitude to the hypotenuse: b im the figure, arabe |. — i ——— We can wnite the area of a right tangle m two ways: A(AABC Equatng and simplifying gives us a- h, = b -c. The legs of a night triangle measure 7 cm and 24 em. Find the height drawn to the hypotenuse of tis triangle. Let the hypotenuse be a. By the Pythagorean Theorem we have a’ = 7° + 24 = 49 + 976 = 629, ie. = 29cm. 7.24, 10. h, 168 25 Asan exercise, try solving this problem using only the formulas we studied m the previous section, Can you do it? Now using ah, = bc gives us 25+ h, If the base lengths and heights of two triangles are the same then their areas are equal. base, -height, 2 Let the area of the frst triangle be A, a, = bse heioht, 2 Then A, = A,, as required. . and the second area be (because the bases and heights are equal), In the figure, A(ABC) = 112 em? DE = 12 mand AH = 8 emare given Fmd BD = EC =x Avoa of Polygon Reson n Proof Avrat of Triangular Regions In the figure, AH ‘8 em is the common height of AABD, AADE and AEC. ‘The base lengths BD = BC = x are the same and their heights are also equal, so by Property 2. we have A(AABD) = A(AAEC) = S. DE:AH_ 128 ey Also, A(AADE) From the figure we can say A(ABC) = A(AABD) + A(AADE) + A(AAEC) H2=S +4848 2S = 64, $= 32cm" Now, since A(AABD) = ‘A median of a triangle divides the area of 7 the mangle mto two equal parts: m the figure, if BD = DC then A(ABAD) = A(ADAC) Let AD be the median and AH be the alutude, as m the figure, So BD = DC. We can say that AH is the altitude of both of the triangles ABD and AADC, so BD AH ana acaabc) = PCAN. A(AABD) = BD-AH _ DC-AH But since BD = DC we can write which means A(AABD) Conclusion By applying Property 3 repeatedly, we can see that a tangle can be divided into equal parts by taking successive medians For example, if BD = DE = EF = FC in the figure opposite ten S, BO the scure shows a trang ABC. BD is the 7 Solution Proof ‘median of side AC m AABC, DE is the median of side BC in ABCD, and EF is the ‘median of side BD in ABDE. Given that » A(QDEF) = 5 om’, find A(AABC). In ABDE, BF = ED so A(ABEP) = A(ADEF) = 9 cm and so A(ABDE) = 10 cam’ In ABCD, BE = EC so A(ACDE) = A(ABDE) = 10 em and so A(ABCD) = 20 em* In AABC, AD = DC so A(AABD) = A(ABCD) = 20 cm and so A(AABC) = 40 cm*. If the heights of wo ianges are the same then the ratio of the areas is the same as the ratio of their base lengths: in the figure, ‘Tangles ABD and AADC m the figure both have common alntude AH AH-BD A@ABD) _~ A(AADC: ‘DC So ES 2 ance A(ABD) _BD_m aneeling common terns ges Us 5p) ~ De Note ‘We can use the letter S to mean the common muliplier In triangle rao problems whieh use Uns property: A(AABD) = mS and A(AADC) = nS. Avoa of Polygon Reson En 31 Ex 32 En 33 Solution Arras of Pangular Regions In the figure, A(AABC) = 70 em", ; BD = 4 cmand DC = 10 emare gwen. Find A(AABD), By Property 4 we can wnte A(ABD) = 4S and A(AADC) = 10S. Since A(QABC) = A(AABD) + A(AADC), we haye 70 = 45 + 105, 1¢. 145 = 70 and so S = 5 cm’ So A(AABD) = 4S = eto 10 c +8 = 20 om¥ In the figure, AE = 5 em, EB = 3em, Z BD = 6 cmand DC = 5 cm, The area of AAED is 15 em*. What is the area of ABC? In AABD, AE = Sand EB = A(AAED) = 9S and A(EBD) = 35, ISS 58= A(QAED) + A(EBD) = 95 +35 = 85 = 8-3 Since BD = 6 and DC = 5 we can write A(AABD) = 6X and A(AADC) = 5X. So A(AABD) = 6X = So A(AABC) = A(AABD) + A(AADC) = 6X + OX = LIX = 11-4 = 44 em’. so we can write 5. Le, $ = 3 om’ 4, 1.e.X = 4m’, In the figure, ABCD 3s a rectangle. DE = EC, AF = FG = GB and 7 A(QEFG) = 6 cm” are given. Find A(ABCD). ' \ id 7G 8 Let us draw EA and EB. By Property 4 A(AEP) = A(AEFG) = A(AEGB) = 6 cm* So A(AEAB) = 6 + 6 + 6 = 18 cm’. (ABCD) 2 We also know that A(AEAB by the properties of a rectangle. So A(ABCD) = 2 - A(AEAB) = 2-18 = 36 cm [EQUEE 34 othe nate, 8c is a rectangle. DC is bees Proof divided mto five equal parts and AB 1s. F divided into four equal parts. Given that A(ABCD) = 180 em, find A(JGF) Let us divide IGF ito two tangles and find yg es the area of each triangle. We know that if we comnect any pomnt on one side of a rectangle to the two non-adjacent vertices then the area of the triangle formed will be half the area of the rectangle. Let us draw the nes GI, DI and CI. ‘A(ABCD) 2 ‘The base DC of ADIC 1s divided anto five equal parts, so by Property 4 we can waite A(ADIC) = 0S By the properties of a rectangle we have A(ADIC) 0 cm So A(AIFG) = $ 2 = 18 em’. In the same way we can draw GA and GB to get ACABCD) _ 180 _ A(aaBGy = AABED) _ 180 _ 99 om* 2 2 Since AB is divided into four equal parts, we have ACAI]; 205 =m" : Ply Ace) = Aa + aac = 18-+ If the bases of two triangles are the same a then the rato of their areas is the same as the ratio of their heights: in the figure, BC 1s.a common base, so BCI, A(AABC) AGDBC) BCT 2 A(AABC) Canceling common terms gwes us AAO _ 5 required. anes US ACADBC) Aves of Pelygonal esions [=u 35 Arras of Pangular Regions In the figure, BC = 8 mand DH = 4m, Given that A(ABDC) = 20 cm’, find AD. Let AD = x, then AH = x +4 a We have (SABC) = A(ABDC) + A(ADBC). A(AABC) _ AHL By Property 9, “070 — Ag A@DBC) DIT Check Yourself 6 1. The legs of a right triangle measuze 9 cm and 12 em. Find the length of the altitude to the hypotenuse of this tnangle, 2. Two parallel lines are given, Two points A and Bie on one of the lines and pomts C, Dand E he on the other Ime, What can you say about A(AABC), A(AABD) and. A(AABE)? 3. In the figure, AABC 4s an 1sosceles triangle with a AB = AC. Given that AD = DE = EF = FG = GC 2 and AH 1s the altitude to de BC, sna SOPEH). é ‘AGABC) , ( e u ¢ 4. Im the figure, BC is divided into four equal parts and A AC 1s divided into three equal parts. If A(AEGH) = 4 cm’, find A(AABC). a ses F iG Proof Ex 36 Solution 5. In the igure, ABCD 1s a rectangle. AB is divided into four equal parts and DC is divided mto three equal parts. IA(ABCD) = 120 em’, find A(EGIH) Answers 1 89 om 2. they are equal 3 1 B 10 The rato of the areas of the two triangles a } formed by the bisector of an angle in a triangle 1s the same as the ratio of the lengths of the two sides separated by the bisector: im the figure, In the figure, let us draw perpendiculars from the point N to the sides AB and AC and let the intersection points of these perpendiculars and. the sides be D and E respectively ‘Smncea point on the bisector of an angle is the same distance from the two sides of the angle, ‘we can write ND = NE = x. AB-ND jo ACA 2 AB A(GGANC) AC-NE AC.x AC b as required. In the figure, CN 1s the bisector of ZC. Gwen that AC = 4em, BC = 9 cmand js A(AANC) = 12 em, find A(AABC), By Property 6 we have M@ANC) _ AC ‘A@QBNC) BC m4 AGBNC) 9 So A(AABC) = A(AANC) + A(SBNC) = 12 + 27 = 39 cm and so A(ABNC) = 27 cm Avoa of Polygon Reson ‘The medians of a triangle together divide the area of the triangle into six equal parts. - AX —27 em, Finally, A(DHIF) = A(ADFH) + A(QFHD = 36 + 27 = 63 cm. GQ tothe teure, ABCD isa rectangle Given that , CE 1. BD, BE = 9 em and DE = 16 em, find A(ABCD). By using the mete relations in a right & mangle in ACDB we have i DE BE, ie. 16-9 and CE = 12 em, A(ABCD) 2 So A(ABCD) = 2- A(ABDC) = Metzic relations in a | CE right eriande ce’ Also, A(ABDC) = Prove Uiat A(GABC) = 2A’ sin A. sin B - sin C, where R is the circumadius of AABC. a_b_e By the law ofsines, = = 5G 2Rsind-2Rsm B-2Rsinc aR =2R, Le. @ = 2R sin A, b = 28 sin Band ¢ = 2R sinc. A. sin B sin C, as required. Arras of Pangular Regions 4G inne figure, SABC and SCDE are two : tangles and mZD = 90°, Find A(AABC) By the Pythagorean Theorem in ACDE, CE = 9° + 12" = 169, 16. CB = 13 aie ZACB and ZDCE are vertical angles so they / are equal. So sin(ZACB) = sm(ZDCE) 3s Finally; by the trigonomeme formula for the area of a tangle we can write 1 1 8 A(AABC) = >. AC -BC -sin(ZACB) = —.8 13° = 20, 2 2 B GB inthe nue, ABC i a right triangle with ‘mZC = 90°. Given that AD 1s an angle bisector, AB = 6 cm and DC = 3 cm, find A(AABD). Let AC = hand BD = x By using the bisector theorem we have AB_AC |, 6_h Bak te. BD DC xh=l8 Finally, A(AABD) 4G inthe hire, sABC is a right triangle ; Given that A(ABD) = 1 cm’, A(AADC) = 9 em" and AD 1 BC, find the . length AC = x Avoa of Polygon Reson Let AD = h, BD = mand DC By the metric relations in a right triangle we have h* mand Asano = 9 =", Also, A(AABD) which give us m= 2and n-h= 18. ‘Muluplving these equations gives us (m li) (nh) = 2-18, ne. ms neh 36h = JE cm. But we know h® =m . ABCD then A(APAB) + A(APCD) = A(APBC) +A(APDA) _ (ABCD) —sT eee Proof Let us draw the altitudes PE and PF from a pOIt P to sides AB and CD respectively. ¥ Then PE + PF =h, i D a Also, A(APAB) + A(APCD) = a zs a ah, - 2 A(ABCD) _— In the same way we can prove that A(APBC)+A(APDA) SENET GO tren neue, Pia yout msde the ; 3 parallelogram ABCD. Given that A(APAB) = 13 om, A(QPCD) = 12 em’ ang AAPBC) _ 2 ‘ACAPDAY find A(APBC), A(APDA) and A(ABCD). Solution Weare gen A@PBC) _2 55. we cam wnite A(APBC) = 26 and A(APDA) = 35. Acapbay 3 By the previous theorem, A(@PAB) + A(APCD) = A(APBC) + AcapDA) = ACABCD) 2 13 + 12 = 98 +35, ie. 88 = 25 and S = 5, Areas of Polygon Region So A(APBC) = 28 = 2-9 = 10 cm, A(APDA) = 38 = 3-5 A(ABCD) = 2 - (A(APAB) + A(APCD)) = 2. (13 + 12) = 2-25 = 50 em, 3 em and Gen 1. Any diagonal in a parallelogram divides it nto two equal parts. eae ae Tectia ne : yurldegan tthe poate ees and 4, Any pomt E on any side of a 7 parallelogram which is connected (0 the two non-adjacent vertices creates a triangle which has half area of the parallelogram: m the figure, A(ABCD) é a “S A(AABE) = 9. Connecting the midpoints of opposite sides of a parallelogram ereates four congruent parallelogram, F Areas ef Quadriatrals G1 inthe igure, ABCD isa parallelogram and 2 ¢ Es the midpoint of side AB. Gwen that DE = 6 em, AD = 8 emand % m(ZADB) = 60°, find the area of ABCD. A : a Solution By the trigonometnc formula for the area of a tnangle, 1 1 819 F om" A(AADE) = >.6.8.singo"= 4.6.8 z 2 By Property 10.3 we can now write (ABCD) = 4+ A(AADE) = 4-128 = 48 V3 cm’, (ESHEETS GQ weve neure, anc a parallelogram and E ; z 184 pou on side AB Given that DE = 6 em, CE = Temand mm(ZDEC) = 90°, find the sum of the areas of the shaded regions. él G 3 Solution By Property 10.4, A(aDEC) = “ABCP) A(ABCD) and so the sum of the areas of the shaded regions 1salso So the wm ofthe areas of he shaded reponse AABCD) = A(ADEC) = GB the ogure, ABCD tsa parallelogram. Given = that AE = EF = FB, DG = GH = HI= ICand A(ABCD) = 120 cm quadniateral EFIG. find the area of Let us comnect pout G to A, B and P. ‘Then we can find A(EFIG) as the sum of A(AEGF) and A(AGFI) By Property 10.4 we have A(AAGB) Aas 120 60 exit In AGB, the base AB is vided mto three equal parts. So Asker) = AASB) 90 emt ‘Now let us connect point F to D and C. Areas of Polygonal Regions EDS 64 Solu EEE 65 Areas ef Quadriatrals A(ABCD) _ 120 By Property 10.4 we have A(ADFC) =“ 5780 em" In ADFC the base CD 1s divided into four equal parts and so A(AGFI) = 2 —_ 2-20 om’ So the sum of the shaded areas is A(AEGF) + A(GFI) = 20 + 30 = 50 em* Im the figure, ABCD 15 a parallelogram. E 15 a 3 the midpomt of AB and BD 1s a diagonal. Gwen that A(ABEF) = 8 cm’, find A(ABCD) a " 4 I ABCD 1s a parallelogram then EB || DC. So m(ZPBE) = m(ZFDC), m(ZPEB) = m(ZFCD) and m(ZEFB) = m(ZDFC). So ABFA ~ ACED and tne rano ofsimstaty sk 2B = 1 1 FB_I we k= 2 mon FE Levusdiow the segment DE. in abe, 2} and aca) = 8 ot So A(ADEF) = 2. 8 = 16 cm" and A(ADEB) = Finally, by Property 10.3 we can wnite A(ABCD) = 4» A(ADEB) = 4.24 416 = 24cm" 6 cm", ABCD 3s a parallelogram and E and F are the mdpomits of sides AB and BC respectively. Given. that A(DEFC) = 30 em’, find A(ABCD), Let us draw the line segment DF. By Property 10.3 we can wnite A(ABCD) an or Dy Proper 10.9 we can wate wcabeD) A(ABBF) = 4 = ACABCD) 56 acapnpy = MABCD) _AABCD) 3. gagcpy 7 > ea Now A(DEC)= A(ADEF)+A(aDFC) =2°MABCD) , MABCD) _9:AUABCD)_ 99, 30 which gves A(ABCD) = Check Yourself 10 1. Two sides of a parallelogram measure 14 cm and 18 em. Given that the acute angles in Uus ‘parallelogram measure 49°, find the area of the parallelogram. 2. Im the figure, ABCD 1s a parallelogram and » 13 i ‘m(ZDBC) = 90°. Given that BC = 5 em and DC = 13 em, find A(ABCD). A 5 8. In the figure, ABCD 3s a parallelogram, » A ‘ Given that m(2D) = 120°, m(ZBEC) AE = 5 cm and DC = 9 em, find ABCD) ‘5 E a 4. In the figure, ABCD 1s a parallelogram and Ps. v, ¢ omit inside it, The area of the parallelogram is, 76 cm’, Find the sum of the areas of the shaded regions. a % ©. In the figure, ABCD is a parallelogram and side DEPGH I ¢ DC 1s divided into six equal parts. Given that A(ABCD) = 150 cmt, find A(ABIG), 5 In the figure, ABCD 1s a parallelogram. Side AD 4s divided imto five equal parts and side BC 1s divided into four equal parts. Given that A(ABCD) = 200 em*, find A(GIKE), Areas of Polygonal Regions Tn the gure, ABCD is a parallelogram, E 1s the » F amtersection point of its diagonals, and BF 1s se oe perpendicular to AC. Given that BF = 3 em, DE = 5 emand CF = 6 em, find the atea of the a: parallelogram ABCD. Answers 11269em' 2 60cm? 3 18/3em* 4. 38cm? 5 25m? 6 90cm 7 60cm C. AREA OF A RHOMBUS A thombus is also a parallelogram, so it shares the rules and properties that we have seen for parallelograms. It also has some additional properties. If ABCD 1s a thombus then the following statements are true. 1. The area is given by A(ABCD) = base x height: coerce [ Aan | sides and parallel opposite eid 2. A(ABCD) = a®- sin A 3. Since the diagonals of a thombus are perpendicular to each other, AC-BD-sin90° _AC-BD 2 A(ABCD! 6G Ashombus has sides of 6 cm. Given that its height i 4 em, find the area of this rhombus. Solution A(ABCD) = base x height = 6 x 4 = 24 em. ERNIE G7 the dagonats ofa mombus measure 12 cm and 16 cm. Find the length ofits altitude. Solution Leta be the measure of one side of the D shombus. We know that the diagonals of a thombus biseet each other, so in the figure, DE = BE = 6emand AE = EC By the Pythagorean Theorem, 6 + 8°=a'1ea = 100m, AC-BD _ 16-12 96 cays 2 2 Since A(ABCD) = a h, we have 96 = 10- h, 1.0. h, = 9.6 om. Also, A(ABCD) Areas ef Quadriatrals GB the acute angle ofa shombus measures 30° and its height is 6 cm, Find its area. Solution Leta be the side length. In the figure, DE 1s, the alttude and DE = 6 em sin30°= PE gg 1-8 hy 2D D 6 AD =a = 12cm, a : 7% So A(ABCD) = a « h, [ESE 69 1re tonger diagonal of a rhombus measures 8 cm and the obtuse angle of the rhombus measures 120°, Find the area of ths shombus, Solution Look at the figure. We know that the diagonals of a rhombus are perpendicular to each other and bisect each other. Also, the diagonals bisect the vertex angles. Consider the trimgle ADEC in the figure AC = 8 em s0EC = 4 em, and by using trigonometric ratios DE ¥3_DE 4 tan30°= Fe SP = DE= So by Property 11.3, A(ABCD) = _AC-BD 2 EXE 7.O wine neure, a8c0 ws a thombus, Given . that BK and CK are angle bisectors and A(ABCK) = 8 cm’, find the area of the rhombus. Solution We know that i Z ‘m(ZABC) + m(ZBCD) = 180°, so ‘m(ZKBC) + m(ZBCK) = 90° and m(ZBKC) = 90 ‘We also know that the diagonals of a rhombus are perpendicular to each other, s0 Kis the intersection point of the diagonals. and so A(ABCD) = 4.8 = 32 em‘ By Mogens 102 we can wre acc) = ACABCD) Areas of Polygonal Regions EEUEE 7 1b the ngme, a8cD 8 a zhombus Gwen that AE = 3-EC, EF LDC, 3.DF FC and EF = V3 cm, find the area of the rhombus. Solution AE =3-EC so let EC 3. DF = 8 FC solet DF = Sy and FC = 3y cand AE = 3x Now let us draw the diagonal BD and mark the intersection pount O ofthe diagonals. AC = 4x 90 06 = 2x, Im(ZEFC) = 90" = m(ZDOC) and ZOCD is a common angle, so m(ZFEC) = m(ZODC) and AEFC - ADOC. BFLEC FC ,, x po be™ oc"! ay ae By the Pythagorean Theorem im AEFC, (By + Gy? =x and x = 23 which gives us OC = 4/3 em, AC = 8¥3 cm, DC = Bem. By the Pythagorean Theorem in ADOC, DO® + OC* = DC’, so DO* ~ 4By AC-BD_ 85-8 z BS aayf om By similarity, and sox" = 12y7 124), $0 64 — 48 = 16,1.e, DO = 4emand BD = 8 em, Fmally, A(ABCD) Check Yourself 11 1. One of the diagonals of a thombus 1s the same length as one side of the rhombus. Find, the area of the rhombus if one side measures 8 cm. 2 The aluude BH 1s drawn from vertex B of a rhombus to side CD. Given that DH = 2 em and CH = 3 cm, find the area of this shombus, 3. A rhombus has perimeter 80 cm. Given that one of the diagonals has length 24 cm, find the area of the rhombus, (One side of a rhombus measures 18 cm, Given that an obtuse angle in the rhombus measures 150°, find the area of the rhombus. Areas ef Quadriatrals 5. The diagonals of a rhombus are 25 cm and 30 em long, Find its area. 6. The lengths of the diagonals of a rhombus have ratio 3 ; 4, Given that the area of this thombus is 216 unt’, find the lengths of the diagonals and one side of the rhombus, > 7 In the figure, ABCD 1s a rhombus and A(ABCD) the miépomt of BC. Find A(AABF). 120 em’, Bs, Answers A 1323em® 2.200m® 3. 384em? 4. 162em* 5.375 em* 6. diagonals: 24, 18; sidelength: 157-20 em? e D. AREA OF A SQUARE A square 1s a special type of rectangle, so we <7 can fund its area by muluplving the lengths FR P of ts two adjacent sides: mm the figure A(ABCD) = a-a, 16 SF A(ABCD) = a? YN The diagonals of a square are perpendicular to each other. Also, if we say that the length ofa diagonal is e then the area of the square 1s ACABCD) = A square 1s also a parallelogram, so it shares all the properties of a parallelogram. EZUEIES 7Q Fina the area ofa square whose perimeter is 28 em Solution The perimeter is 4. a = 28, soa = 7 em, So the area is a° = 7" = 49 em’ JB tne rene ofthe diagonal ofa square ss 19 em, Pind the aea of ths square, 4 . Solution tes thetength of the diagonal then A... = =H rem" Areas of Polygonal Regions EEZNIE 7.4 4 suare has area 20 em? ind the long ofits diagonal I the area isa” then 20 = a and soa = V0 = 25 em. The length of the diagonal is therefore e = aJ3 = 25. V5 = 2Vi0 om Solu (FEQUEES 75 tothe nur, ac isa square 0, © Given that AE = 3 emand EB = 5 em, find the area of the shaded region, Since AB = AE + EB, we have ASE 5 B a=345=8em, So the shaded region has area A(ABCD) ~ A(AADE) = 8° — 7G whe neue, ABCD 1s a square and AAEC 18 b c equilateral tnangle. Given that A(QAEC) = 9/3 cm’, find the area of this | Solution Let one side of the square be a and let the E ceece 5 The ara A of an| So A(AAEC)=*S™ i.e, oy = 2S", equilateral triangle 4 4 with side ngth @ is Smee ABCD 1s a square, A(ABCD} 77 tthe figure, ABCD sa square A ‘ Given that DE = 4 mand ‘m(ZAEC) = 120%, find the area of the square. Eb oo Areas ef Quadriatrals iis From the figure we can see m(ZDEC) = Let one side of the square be a, so DC = a Aliso, tan 60° = P© whieh gves us DE v3 em. = (413) = 48 em". So A(ABCD) EZUEUN 7G m tne figme, ABCD is a square, Side AB is 3 c divided into three equal parts and side DE 1s divided into four equal parts. Given that the area of the shaded region a5 3 cm*, find A(ABCD), Let us draw the Line EC, A(ACHD = 3s given, so A(QDEC) = 4.3= 12.4 From the properbes of a parallelogram we know that A(ADEC) = ABCD) So A(ABCD) = 2-12 = 24 em’. 79 wine ngure, ABCDs a square. i A Given that AD = EC, EB = 8 em and. ‘m(ZAEB) = 90°, find A(AEBC) ‘i a" » Solution We know that EC = AD = BC, so AEBC is an 4 sosceles triangle, Let us draw the altitude CH m AEBC. Since AEBC is isosceles, CH is also a median so EH = HB = 4cm. Let m(ZHBC) = x, then m(ZEBA) = m(ZBCH) = 90° x and m(ZEAB) = x. m(2CHB) = m(ZAEB) = 90° and AB = BC, so by the ASA Congruence Theorem we can say AAEB = ABHC and CH = EB = 8 cm EB-CH 8-8 2 2 2 cm* So A(AEBC: Areas ofPelygoral eons Check Yourself 12 1. A square has area x cm* and perimeter x cm. What ts the value of x? 2. The perimeter of a square 1s 8 cm. Find the area of this square. 3. Im the figure, ABCD and EFGH are wo z squares, Given that sum of the perimeters H of these squares is 36 cm and area of the shaded region 1s 27 em’, find the areas of 6 the two squares. 4. The diagonal of a square 1s 8 cm long. Each side of the square is extended by 1 em. By how much does the area of square merease? 5. The penmeter of a square ABCD and the perimeter of an equilateral mangle EFG are AGEPC) Squad Find caBCDy 6. In the figure, ABCD is a square with side length 12. em, The side AB is divided into four equal parts, BC is divided into two equal parts, and DC and AD are each divided into three equal parts. Find the area of the shaded region. E. AREA OF A TRAPEZOID A epez0id is a) quadeiateral with two parallel side. Areas ef Quadriatrals Answers Vi odour 3 openioenr sa edew ME a aren The wat ofa penn ais a => core : (ABCD) = **°. i _. —————~—_——— Proof Look at the figure. Let us draw the alntudes ps ¢ 4 DK and BH ‘We know AB || DC, so DK = BH 8 * Now we draw the diagonal BD. So A(ABCD) = A(AABD) + A(ABCD) A, a BO trine hee, asc sa trapezoid jane AB = 13 em, DC = 3 em and CH = Gem are tiven. Find the area of this trapezoid Solution Weare given a = 13,¢= 3 andh = By the formula for the area of a trapezoid, A(ABCD) = = hae, A(ABCD) = 13+3 6 =8-6 Note ‘We know that the Line which comnects the midpoints of the legs of a trapezoid 1s called a median, and dhe length of the median = © SoAcaBeD) = *°.n = median x height EEZUEES G1 the medion ofa napezoid measures 10 cm and the height is 14 em. What isthe area ofthis trapezoid? Solution Area = median x height = 10-14 = 140 cm’, Areas of Polygon Region GQ seine swe, ABCD isa trapezoid. Given that Solution Ap isosceles trapezoid isa trapezoid mith congrent bee Solution Areas ef Quadriatrals ns BC = 12 em, DC = 8 em, m(ZDAB) = 49° and m(ZABC) = 30°, find the area of ABCD. 4 Fst we draw the altitudes DH and CK. bos In ACKB, BC = 12 emsoCK = h=6 emand KB = 6/3 cm. (30°-60"-90° tangle) In AADH, m(ZDAH) = 45° and AH 1 DH so bs m(ZADE) = 45° = vos So AH = DH and we know DH = CK = 6 em, We also know that DC = HK = 8 em. Soa = AH + HK + KB = 6 +8 + 6/3 = (14 + 6V3) om, and DC = 8 em Fralte ABCD) = 228 = MEO 8g In the figure, ABCD is an isosceles trapezoid, AB = 24 em, BC = AD = 10 cmand DC = 12 emare gven. Find the area of this trapezoid, Furst we draw the alutudes DH and CK to Sai base AB, So HK = DC W2em. w Since ABCD 1s isosceles, AH = KB = (22. +648) -3 =(66 +18 48) em* But AB = AH + HK + KB, which gives A HB 2M =x+ 12 +x,x=6em. In ACKB, BC = 10 cm and KB = 6 om. By the Pythagorean Theorem, CK + KB" = BC’, Wf +6 = 10°, k= 8 em. So A(ABCD} G4. trie feare, ABCD is a neht trapezoid as Given that AB = 8 om, BE = 9 em, BC = 9 cm and DC = 4 em, tind the area of AED. le 3 Solution A(AAED) = A(ABCD) ~ A(AABE) ~ A(ADCE) a a] 8 iy #348 2 2 = 6-8) ~ 12-10 = 26 cm? In the figure, ABCD #5 a right trapezoid and 5 5 Es the midpomt of AD. Given that BC = 6 em and BE = 5 cm, find the area of the trapezoid, 6 Let us draw the median EF. Then BF |) AB || DC, EF L BC and F is the A 2 ‘midpomt of BC. So BF = 3 em, ABFB 1s a night triangle so EB" = EF* + FB, 10. 5° = EF? +3°, EF = 4em Finally, A(ABCD) = median « height = EF BC = 4.6 = 24 em? GG Av isoscetes trapezoid has a diagonal of 10 em and height 6 em, Find the area of this trapezoid, Solution Let us draw the alutudes DK and CH. 7 ABCD 1s am isosceles trapezoid, so let us wnte AK = HB = x By the Pythagorean Theorem im AAHC we have AC* = AH + CH’,1.6. AH = 8 em MAH = 8 and AK = x then KH = DC ABDC oy 844188 “x, AB= 844, So ACABCD) =8-6=48 em® Areas of Polygonal Regions In the figure, ABCD is a trapezoid and O18 the mtersection pomt of its diagonals. If NT A(AAOD) = x, A(AAOB) = m, A(ABOC) and A(ACOD) = n, then vin. ACABCD) = (im + a) Proof | AADC and ABDC have a common base DC and two common altitudes, so A(QADC) therefore x (ABDC). Sox +n =y mand ‘We know that m any quadrilateral, 5,8, =5.- 5, This gives us x-y = m-n. Sox: y = A(ABCD) xtytmin jan eden new m+2- mn tn (dn + Say’ (EQ G7 toe gure, ABCD isa trapezoid and Orsthe a mtersection pomt of ts diagonals. Given that A(QDOC) = 9 em and A(AAOB) = 25 em’, find the area of ABCD. A(AAOD) = ACABOC) : : Z A(G&DOCY-ACAAOB) = 15cm" So A(ABCD) = A(AAOD) + A(BOC) + A(ADOC) + A(AOB) = 1d +19 +9429 = 64 om’ Areas ef Quadriatrals [EQQIEN SG tthe see, asco is a trapezoid and Pis Solution Proof the intersection point of uts diagonals, Given that A(SABD) = 12 em* and A(ABCD) = 8c find the areas of ADPC and APCB. Let A(ADP) = A(APCB) = x, then A(QDPC) = 8 = x and A(AABP) = 12- x By the previous theorem, we have Let ABCD be a trapezoid and let E be the midpoint of leg AD. Then a¢apecy = AARP) Let us draw the median EF and the altitudes CH and KB of the trapezoids DEFC and EABF respectively. Smice EP 1s the median, CH = BK Now A(ABCF) = a0)" ag 8 are h EP-BK_~9 "9 _(a+e)-h AAEBF) i i So A(AEBC) = A(AECF) + A(AEBF) = @*O-R ,(@+0-h 8 ave (ate-h 9 4 2 A(ABCD) = AAACOY as required. 5 1 Areas of Polygonal Regions EE 89 Solution Ear 90 Solutio En 91 Solution Areas ef Quadriatrals In the figure, ABCD is a trapezoid and E 1s Dg the midpomt of leg AD. Gen that AB = 8 cm, CD = 5 cm and e A(BEC) = 26 cm*, find the height of the trapezoid, 4 = a By the previous theorem we can mite A(ABCD) = 2 - A(ABEC) AB+CD aff hare = 92cm" But sce A(ABCD) 1h, we have 52 In the figure, ABCD 1s a trapezoid and F is o c the midpoint of BC. 5 Gwen that AD = 8 cm, EF = 7 cm and AD 1 EP, find the area of ‘ the trapezoid. 4 B Let us draw DF and AF, By the previous theorem we can write AD-EF A(ABCD) = 2- A(MADF) = 2 =8-7= 96 em" ABCD 1s a trapezoid with bases AB and DC and median EF such that E 3s on AD and Fs on. BC. Given that P as the mdpemt of EF and A(AAPE) = 4 em’, find (ABCD). Let the lengths of the bases be a and cand a c Jet the height of the trapezoud be h fee N\ Then EF =4*°, f a : FR Ep = 2+ and Pi 7 a A By O ———— ate So A(AAPE) SP-PH - MABE ‘This gives us A(ABCD) = 8. 4 = 32 en En 92 EES 93 In the figure, ABCD is a trapezoid and KL || AB || DC. Gwen that A(ABLK) AB = 8 cmand DC KL=x. (KLCD), 6 cm, find the length Let KL = x and let us draw line segment MN parallel to DA, as show m the figure. So KL = AN = DM =x and NB = 8~x, cm = x-6. Now let us draw the heights f, and f from point L to the bases DM and AB, respectively Is given that A(ABLR) = A(KLCD) +e, Ste hate Be St ae, w 2 2 5 h 64x In ACLM and ABNL, m(ZCLM) = m(ZBLN), m(ZLCM) = m(ZLBN), and m(ZLMC) = m(ZLNB). ‘This means ACLM ~ ABLN, 1¢. In the figure, ABCD 1s a trapezoid. Base AB Fie is divided into four equal parts and base DC 1s divided into three equal parts. Given that AB = 2- DC, find ACEGIH) ‘A(ABCD) Areas of Polygonal Regions Let DC = x so AB = 2x, and et h be the height of the trapezoid. Let us draw the line segments El, ED and EC. We can say A(ADEC) = Dit = HI = ICs given, so A(AHED AO) Nove let us draw the line segments JA and 1B We can wnte ACAAIB) = 724 =x AB = EF = FG = GB sven, so A(a£1G)= AOA) _ oh, ah xh _ uh So A(EGIH) = A@QHED + A(aBIG) = + Sh A cod Finatly, AEGIHH), ACABCD) 94 twine ngute, ABCD 3 a trapezoid. Given that AD EC and EB = 2. AB, fnd ACAEED) ACABCD) b f Let AE = x, then EB = 2. AE = 2x, and DC = x because AECD is parallelogram. pe_x_l _ _ Epo ayy SORF =2-CP and BF = 2. DF. so if A(ADPC) = S then A(ABFE) = 4S. Since DC || EB, ADFC - ABFE and A(QDFC) _ Aye _ 1 AQBFE) 2’ 4 IFA(ABFE) = 45 then A(ABFC) = 25 because EF = 2. CF Now let us draw DE. In ADEB, BF = 2. DF and since A(ABFE) = 4S, A(ADEF) = 25, A(ADEB) By Property 8, ofthe areas i oqusl to the square ofthe ratio of sma In AABD, EB = 2-AE and A(ADEB) = 45 + 25 = 68, so A(aaDE) = 3CO 3s, A(AEFD) _ A(AADE)+A(ADEF) _35 +25 _ 5 © “ACABCD) ‘A(ABCD) ws 12 Areas ef Quadriatrals Check Yourself 13 1. The bases of a trapezoid measure 9 cm and 9 em. Given that the height of this, trapezoid is 6 cm, find its area. In the figure, ABCD is a trapezoid. j—>— Given that AD = 48 em, DC = 8 em, 4a m(ZADC) = 120° and m(ZABC) = 45°, ‘ind the area ofthis trapezoid ! . 3. ‘The length of the shorter base of a right trapezoid measures 6 em. The length of the Ionger base 15 12 cm and one of the base angles measures 60°, Find the area of this trapezoid. 4 Im the figure, ABCD is an isosceles trapezoid, Given hat AB = 10 em, BC = 4 em and fo m(2BCD) = 120°, find the area of the trapezoid, n % 2 9. In the figure, ABCD isa trapezoid and E isa psc point on AB. Given that AE = 4 em EB =6cmand De = 5 em, ng AOE ’ \ . © The ratio of the lengths of the bases of a trapezoid is 6 : 13. The height of the ‘trapezoid 1s 20 cm and its area 1s 380 em*, Find the length of the longer base. 7. Im the figure, ABCD is a uapezoid and EF is tsp, id c median, Given that AB = 3 em, DC = 19 em and A(ABCD) = 90 cm, find A(ABFE). p é -s 8 8. ABCD is trapezoid and O is the intersection pont ofits diagonals. Given that A(AABC) = 18 cm* and A(AACD) = 12 em*, find A(AAOD). Areas of Polygonal Regions 8. In the figure, ABCD 1s a trapezoid and its diagonal BD 1s, psc ‘aivided mto three equal parts. Gen that AB = 24 em, DC = 8 em and A(AAEF) = 12.em*, ‘find the area of the trapezoid. Answers 4 a L adem’ 2.666 + 6,9) em? 9.94/3em" 4 168em' 5 6 260m 7 30 em" 8 Sem? 9. 48 em" 5 F. AREA OF A KITE E ee eee the diagonals ofa (gunéeitera are Perpendicular to each beaee then the a of the quadniaterlc siascp) = 22-82 En 95 Solu [EO 96 Solution Areas ef Quadriatrals Recall that a kate is a quadrilateral that has two pars of adjacent congruent sides. The dagonals of a hute are perpendicular to each other. so the area of a kite is half the product of ts diagonals: in the figure, AC-BD (ABCD) = ‘The diagonals of a kite measure 12 cm and 8 cm. Find the area of thas hate AC-BD 12-8 gg oy? 2 2 In the figure, ABCD is a bute. Given that BC = 5 em, DC = 12 mand ‘m(ZBCD) = 90°, find the area of the kite We draw the diagonal BD to get two triangles SABD and ABCD. Since a kite 1s symmetric about its main diagonal, A(QABD) = A(ABCD) and m(ZBAD) = m(ZBCD) = 90°. 512 So A(ABCD) = 2-(ABCD) =2 =60 cm’. (EOE 97 tothe pure, ABCD is a hie i Given that AB = BC = 17 em, AD = DC = 25 emand BD = 28 cm, find the area of the kite 2 Solution Lecus draw the diagonal AC and let the intersection point of the diagonals be O. a Let AO = y and BO = x, then OD = 28x. By the Pythagorean Theorem in ADO we have 29° 28 -» Rearranging this expression gives us Sumulazly, mn AABO we have y° = 17° Le. y = AO = 15 mand AC = 2- 15 = 30 cm. AC-BD 30-28 2 a So ACABCD) 10 em* Check Yourself 14 1. A kite has an area of 120 cm’, Given that one of ts diagonals measures 24 em, Ond the length of the other diagonal, 2 Im the figure, ABCD is a kite and 0 1s the intersection pomt of its diagonals. Given that AB = AD, BC = DC, AO = 9 emand OC = 16 em, find the area of the kite f NI 3, Im the figure, ABCD 1s a kate Gaven that AB = AD = 4 cm, BC = DC = 8 cm and m(ZADC) = 60°, find A ss the area of the kite. Answers 110em 2. 300em* 3. 16/3 em’ Areas ofPelygoral eons CLAM Chau mr aaa The Pythagorean Theorem is one of the oldest and most famous theorems in the history of geometry. Although the theorem was known by the Babylonians and the Egyphians about 1000 years before the time of Pythagoras (who was bom in around 579 BC), Pythagoras was the first person to publish a deductive proef, which is why the theorem was given his name. ‘The Pythagorean Theorem states that the sum of the A squares of the legs ofa right triangle is equal ro the length of ts hypotenuse, 1. [ea where c 1s the hypotenuse and a and b are the legs of the night tangle. Mathematicians since the time of Pythagoras have studied this theorem, and it has been proved in different ways 1n different branches of mathematics. A waiter called Elisha Scott Loomis once published a book with over 360 different proofs of the theorem, Here are four popular proofs. Proof f Let us draw a square ABCD with side length c In the figure, AABE, ABCF, ACDG and ADAH are congruent right iangles inside ABCD with sides a, b and ¢ We can see that the triangles create a smaller square EFGH with side length a —b. Now we can write the area of ABCD mn two ways: A(ABCD) = ¢ and A(ABCD) = 4 - A(AABE) + A(EFGH), Haw =2ab +a" ~2ab + Simplafang this gives ¢ = a? 4b? Proof Let us draw a square ABCD with side 4 I length a + b. Then we choose the pots E, F, G and H such that AEAF, AFBG, AGCH and AHDE are right triangles with sides a, g le bande We can say that each of these four triangles, has hypotenuse c. Proof 4: Now we can wnite A(ABCD) 1m two different ways: A(ABCD) = (a + bY and A(ABCD) = 4. A(AEAP) + A(EFGH). ab +e, so nano) ") (a+b) =2-(°) + TS, which gives us “RP = Canceling the denominators and 2ab gives us a° + b* Let us draw the nght trapezoid ABCD with bases @ and band height a. We can say that BC = DB = c, MEDC = ABAD by the SSS Congruence Theorem. From the angles we can get DB 1 EC. Now we can wnte the area of the right trapezoid m two ways: um of the bases height 2 and A(ABCD) = A(BCDE) + A(AABE). A(ABCD) ath ce ,(a-by-b 2 2 Canceling 2 from both sides and taking ab — bo the left-hand side gives usa” + b* So A(ABCD) = A. Area of a Quadrilateral L. In the figure, Rae De =6, AD =8, 2 1 AB = 10, | BC = 12and > m(ZADC) = 90°. : Find the area of the quadnlateral ABCD. 2, ABCD is a convex quadrilateral and £ is the intersection pommt of ts diagonals, Given that AE = 2, BE = 5, CE = 6, DE = 10 and BC = 5, find the area of ABCD. 3. ABCD 15 a convex quadnilateral and E 1s the Antersection point of its diagonals, DE = 3 em and a AADC) BE = 12 cm are gwen. Find AADC) ‘A(ABCD) 4, In the figure, Es the intersection » punt of the diagonals of ABCD and AC 1s perpendicular to BD. "| Given that BC = 4, DC = 8, AB = BC and m(ZACB) = 30°, nd the area of ABCD. 6. ABCD 1s convex quadmiateral with AD = DC = 8, BD = 14and m(ZADC) = 60°. The angle between Ms diagonals 1s 90°, What 1s A(ABCD)? Areas ef Quadriatrals 6. In the figure, ABCD 18 a > convex quadniateral and Exs the mtersection point of its diagonals. Gwen that DC = BC, m(ZDCA) = 20°, : m(ZDBC) = 25°, AC = Gand E BD = 10, find the area of ABCD. 7. ABCD 1s a convex quadniateral and E1s the intersection pomt of its diagonals. Given that A(ABE) = A(ACDE) = 18 and A(ABCE) = 4+ A(AADE), find the area of ABCD. 8, In the figure, ABCD 1s a © © convex quadnlateral, Eis! the midpoint of DC and F 4s the intersection point of the diagonals. Given that AG = 5 em and aoe Te 6B = 70m, fina A@BCH ‘A(aDAF) B. Area of a Parallelogram Q The sides of a parallelogram measure 10 cm and 24 cm. Given that the length of the altitude to the Ionger side 1s 5 em, find the length of the alutude to the shorter side. 10, Im the figure, ABCD 18.0, a parallelogram, DE 1s perpendicular toABandAPis perpendicular to BC. Given that AB = 12, AD = 6 and AF = 8, find the length DE = x. LL. One of the diagonals of a parallelogram has the same length as one of its sides. Given that the longer side of the parallelogram 1s 6 uns long and its mterior acute angle measures 30°, fnd the area of the parallelogram, 12, Inthe figure, ABCD sy —_ 4 parallelogram and BE 1s perpendicular toc. Gwen thar |) AH =HG=GF=FD, 4 2 A(AGBH) = 6 and EB = 4, nd the length of AB. 13, ABCD is a parallelogram and H is a point on DC such that BH 1 DC. Given that BH = 6, BC = 10 and DH = 8, find the area of ABCD, 14. ABCD 1s a parallelogram and DB 1s its diagonal. Given that m(ZDAB) = 30°, AD 1 BD and BC = 8, fund A(ABCD). 15, Im the figure, ABCD is D c a parallelogram and P is a point inside the parallelogram, 2 A(APAB) = 3 A(APCD), find the area of APAB. 16, One of the diagonals of a parallelogram is 8 units Jong and one of ats sides as 10 umits long, The angle between this side and this diagonal 1s 45° Find the area of the parallelogram. 17, In the figure, ABCD is, a parallelogram, Side DC is divided imo four equal parts and side BC is divided mo" three equal pars, Given that A(ABCD) area of the quadrilateral AICP EB = 4,BC =6 and m(ZDAB) = 190°, find A(AECD) 18. In the figure, ABCD, 1s parallelogram and B is the mid pont of side AB. 19, Inthe figure, ABCD By) yg ote a parallelogram. Side ABs divided into five equal parts and side CD is divided into 4 four equal parts. Given that A(ABCD) = 180, find A(PHK). 20, ABCD is a parallelogram. E and F are two pomts on age AB, and G 18 a pomt on CD. Given that [BP = =-AB and A(AEFG) = 12, find A(ABCD) 21. ABCD is a parallelogram and £ and F are the © mapomts of sides AB and BC respectively. IfG 15 the mtersection point of Lines AF and CE and A(QAEG) = 12, find A(ABCD) 22. Inthefigure, ABCDis y__g__ © a parallelogram and \ sides AD and CD are each divided into Uutee equal parts. t Give that BG LGB, £@ = 6 and GB = 10, fnd the sum of the ateas of the shaded te gions. n C. Area of a Rhombus 23, A rhombus has area 80 anda diagonal which 1s 20 units long, Find the length of the other diagonal. 24, A thombus has diagonals which are 10 cm and 22 em long. Find its area 25. In the figure, ABCD is, > 1 rhombus. Gaven that AB = BD = 8, fnd A(ABCD), é / ‘26. In the figure, ABCD v, c isa thombus and BH isan alutude, Gen 1) that BH = & and fos~ m(ZADC) = 120°, J find the area of this rhombus 27. A thombus has area 200 em’, Given that the length of the altitude to a side is 10 cm, find the measure of the acute angle of this rhombus. 28. na thombus ABCD, the lengths of the diagonals have the ratio 5/2. Given that A(ABCD) = 120, ‘ind the lengths of the diagonals. 20. The sum of the lengths of the diagonals of a thombus 1s 34 and one side of the rhombus measures 13 units. Find the area of this zhombus. Areas ef Quadriatrals 30, 3. 33, In the figure, ABCD isa. thombus and the diagonal BD 3s divided mto sx ‘equal lengths. Given that ABCD) = 72, 4 @ find the sum of the areas of the shaded zegions. In the figure, ABCD 1s a rhombus. Pomt £ 1s on the diagonal BD such that DE = 41 EB = and m(ZDCE) = 90°, ‘Find the area of this rhombus. 2 . In the figure, ABCD 1s, Daz oo a thombus, 01s the intersection point of As diagonals, B15 a point on DC,and @ OE 1 DC. Given that DE = 4, EC = 9 and ‘m(ZOEC) = 90°, find the area of the rhombus. In the figure, ABCD1s_— op i a rhombus and the poimts A, Band F are collinear. Given. that DP 1 BC and 4 so AD = BF = 10, find the area of rhombus ABCD. In the figure, ABCD 1s, a rhombus and the points E and F are the midpomts of sides DC and BC, respectively Given that EF = 10 and AF = 13, find A(ABCD). D. Area of a Square 96, Find the area of the square whose perimeter is 125 units. 36. The diagonal of a squate 6 units long. Find the area ofthis square 37. Inthe figure, ABCD 18a 0 c square and A, Band E are callinear. Given that A(AEDC) = 20, find the area of the square, 38, A rectangular floor has side lengths 12m and 15 m. We want to cover st with square tiles with side length 40 cm. How many tiles do we need? 39. In the figure, ABCD 1s c fa square and E 1s the mudpoint of BC. Given that AE = 946, find the area of Unis square. 40. If we lengthen the sides of a square by 40%, by what percentage will its atea increase? 41. In the figure, AABC 1s ¢ an equilateral triangle and DEFG 1s a square Given that A(AABC) = 4V3, find A(DEFC), a) my 42. Two opposite sides of a square are shortened by 2 units. The area of the rectangle obiained is 39 square umits, What was the length of one side of the original square? 43, ABCD and BCEF are two squares, and P and Q are the respecuve imtersection points of their diagonals, Given that AB = 8, find the area of PBQC. 44. nthe figure, ABCD1s yt c a square and E and F are the midpoints of sides AD and DC . respectively Gwen that AEG) A(ABCG) Sy and S,, find 45, Inthe figne, ABCD isp Fe a square. Side BC is divided into four equal parts and AB is divided into three equal parts. Find the b area of the shaded region sf A(ABCD) = 120. 46, In the figure, ABCD isa c square and ABEC is an equilateral triangle ¥ Given that A(QABE) = 16, find the area of the square, @ 47, Inthe figure, ABCD isa») c square with side Jength 12 cm and AEFG as a rectangle. Given that BF = 3- DF, find A(AEFG) ‘A(ABCD) 48, Inthe figure, ABCD isa square 0 F vith sidelength 10 unuts. Given that DE + EC = 12 and m(ZDEC) = 90°, nd the area of the shaded region ABCED, E. Area of a Trapezoid 49, The two bases of a trapezoid are 12 and 8 units long. Given that the area of the trapezoid 1s, 100 square umits, fd the length of its altitude. 50, In the figure, KLMN is a aright trapezoid. a Given that KL=4,NK = Gand m(ZLMN) = 30°,fmd & = & the area of the trapezoid. 51. In the Ngure, ABCD 1s, a nght trapezoid and E ps is a point on AD. £ Given that AB= 10, DC = 6, AE = 4and ED = 2, find the area a € of AEBC 52, ps In the figure, ABCD is an isosceles trapezoid, Given that AD = BC = 6, DC = 8 and m(ZADC) = 150°, find the area of the trapezoid, Areas ef Quadriatrals 53, sf 8 P ° Jn the figure, PQRS 1s a trapezoid. Given that QR = 10, SR = 6, m(ZPSR) = 135° and ‘m(ZSRQ) = 150°, find the area of the trapezoid PORS. 54, MN and PQ ate the bases of a trapezoid and O18 the intersection point ofits diagonals. Given that A@MON) = 90, MN = 12 and PQ = 8, find the area of trapezoid MNPQ. 55. In the figure, ABCD sp 25 3 an isosceles trapezoid. Gwven that AD LAC, AD = AB = BC, ) aor’ AB = Tand DC = 25, find the azea of this trapezoad, 5G. The diagonals of an isosceles trapezoid are perpendicular to each other. Given that one iagonal 15 20 units long, find the area of Uns trapezoid. 57, In the figure, ABCD 1s », c atrapezoud and E 1s the midpomt of AD. HN Given that EHLBC, FH = 12and BC = 20, find the \ area of ABCD. A 2 58, AB and DC are the bases of a trapezoid and Esa point on base AB. Given that AB = 3» DC and A(ADEC) = 12, find the area of the trapeze. ‘59. In the figure, ABCD isa trapezoid and E, Poand Kae the yy ’ midpoimts of sides AD, BC and AB, respechvely, Given that AEDG, AHFC and AGHK are equilateral tangles and A(AGHK) = 4, find the total area of ‘ x b the trapezoid. 60, In the gure, ABCD is >. a trapezoid and Os the intersection pont of ts diagonals. Given that A@AOD) = 12 and A(ADOC) = 4, find A(ABCD). GL. In the figure, ABCD 1s a Fl atmapezoid and Risa: point on AD. r Given that AB = 13, 1 DC = 7, AK = 10 and A(AABK) = A(KBCD) ‘und the length KD = 62. The two bases of a trapezoid measure 5 em and 15 em, Given that the diagonals measure 12 em_ and 16 cm, find the area of this trapezoid. 63, ABCD is a trapezoid with bases AB and DC. © 4B = 26, BC = 16,CD = 6 and AD = 12 axe given. Find the area of the trapezoid. G4. ABCD 15 an isosceles trapezom with diagonals 12 units long. Given that the angle between a diagonal and a base 1s 30°, find the area of the trapezoid. F. Area of a Kite 65. The diagonals of a kite are 6 and 8 units long, Find the area of this kate 66, In the figure, ABCD is a R hate, AD = AB = 10, w, r Dc=BC=6and ; m(ZABC) = 150°. Find the area of the kate. 67. In the figure, ABCD is, 0 a kute, Given that . AB = AD =6, BC = DC, aR ‘m(ZBAD) = 90"and m(ZADC) = 108°, ¥ find the area of the kite 68. ABCD is a kite. Given that AB = BC, AD = DC = 12, m(ZADC) = 60° and m(ZABC) = 120°, find the area of the kite Areas of Polygonal Regions A. AREA OF A REGULAR POLYGON Recall that a regular palygon is a polygon which is both equilateral (all side lengths are equal) and equsangular (all angles are congruent) ‘excumscnbed eucle, insenbed ecle, apothent All regular polygons can be drawn inside a circle so that all their vertices lie on the curcle. This circle as called the cxrcumsenbed circle of the polygon. In the figure opposite, the larger circle 1s the circumsenibed circle of the hexagon, We can also always draw a cucle inside a regular polygon such that the circle is tangent to every side of the polygon. This circle 8 called the inscribed circle of the polygon. In the figure opposite, the smaller citcle is the nsenibed circle of tie hexagon, The distance between the center of a regular polygon and one of its sides is called the apothem of the polygon. Tn the figure, the length r is the apothem of the hexagon. Note In a regular polygon, the centers of the eircumsenbed and. ansenbed exeles comeide at the same pomt. This point is called the center of the regular polygon. 1. The distance between the center of a polygon and any of ats vertices is the same as the radius of its circumsenbed circle. 2. The apothem of a regular polygon is equal to the radius of its inscribed circle. Note We generally use r to denote the radius of an inscribed circle, and R to denote the radius of a circumscribed circle root of Regular Polygern ara Cites Proof Ena 98 Solutio area-of- e renular-pe ‘The area of a regular polygon is half Une product of ts apothem rand ats perimeter P: ‘We can wnite this alternatively as. ur, where wis half the perimeter of the polygon. Look at the figure. As we have seen, the distance between a center of a regular polygon and any vertex is R, and the distance between the center and a side 38 r Notice that if we connect © with vernees A, B,C, ete. we obtam a number of congruent iangles, Let this number be m The area of each tangle 1s ‘There are m triangles, $0 Avs.ss pene ‘Since the perimeter of the polygon is n a, the expression for its area reduces to 1 As lir-Pu-r, as required 3 a Note ‘We know that in any regular polygon, the tangles that are created when we connect the 60° center of polygon to the vertices are all congruent. So one of the central angles 15. «= ‘This means that we can also find the area of a regular polygon by using trigonometry: if R is, the distance between the center and a vertex of a regular polygon then its area is GRR A regular polygon is given. The radius ofits inscribed circle is 4 em, one side measures 6 cm and its area 2s 96 cm, How many sides does this polygon have? Let the number of sides be n, then the perimeter P = 6n. 1 revue 90 14-0n= tones = 8, She pon hse sie, Areas of Plygorat Regions QQ The radius of the circumscribed circle of a regular octagon is 4 om. Find the area of the Solution ie soe and =— 8 "ae ,48 ; wld 1.00 te ratns of ne mseribed cucle of an equlaterl mange 18 6/3 em. Find the area of the triangle. Solution The centroid of an equilateral mangle and the center of ats mscnibed circle are the same COO me mena atte we snow n= 25, whore a3sa side tngth $0 37-8. 6f8 1846 = om, 16.036 om Since n = 3 we have P = 3a = 3. 36 = 108 cm. So Im an equilateral Jor p= 3645-108 = 32445 om* cing = ar 2 2 EEN 100 7 one site ofa regular hexagon measures 8 em, Find the area of the hexagon, Solution 1 By drawing the diagonals ofthe hexagon we can find the interior angle «= 2° — 60: Look at the figure. We know OA = OB and m(ZAOB) = 60", so E b AAOB 1s an equilateral mangle and R= OA = OB = AB = 8 em. So F + c aR’ sina _6 2 165 cm’ As 8 roar ef Regular Blygerm ara Ciro Solution 2. Asim the previous solution, we can see that AAOB 15 an equilateral triangle. Smee the regular hexagon is made up of six congruent equilateral tnangles, we can wnite A= acat08) <6 6 165 =96 65 em 102 tthe ngure, ABCDEF 5 regular hexagon Given that AD = 6 em, find the area of ABD. F 4 c We can say that AD = 2R = 6 cm, so the side length a = R= 3 em. Let the center of the hexagon be O, then MOB, ABOC, ACOD ate equilateral triangles = with side length 3 em, ip aoe a Also, BCDO 1s a thombus with side length 3 cm. Pp Now, A(BCDO) = A(ABOC) +A(ACOD) 4 40 SWE 3 _ OE be 4 4.2 5vo A(BCDO) “3 _ 93 AcaBbo) = SS cacy Fanally, A(AABD) = A(AAOB) + ACABDO) 2 4 Check Yourself 15 ‘A regular hexagon has area 96/3 cm*. Find the diameter d of its circumscribed circle and its apothem h. Find the area of a regular twelve-sided polygon if the diameter of its etrcumscribed cucle is 8 units, ‘The inseribed circle of a regular polygon has radius 4 em. Given that the area of the polygon 1s 48 cm’ and the length of one side 3s 6 cm, find the number of sides of this Polygon. Aros of Plygora Region with side length 4. Find the area of the | | 4 In the figure, ABCDEF 1s a regular hexagon E > shaded region. ©. In the figure, ABCDEF is a regular hexagon drawn inside a eirele. Given that the radius of the circle is 6, find the area of the hexagon, 6 Inthe figure asa regular octagon figure, ABCDEFGH is a regular octagt r E with side length 6 cm. Find the sum of the areas of the shaded regions, # h ul c L__| 248 3400 41S 8 8a B. AREA OF A CIRCULAR REGION eareutar region: ‘The umon of a circle and its mtenior regon is called a circular region. area of a qurcular region The area ofa circular region 1s the product of r and the square of the radius of the circle: Azo Proof Letus draw twon-gons such that the cucle1s amsenbed mone n-4on and exeumsenbes the cee ee oxher n-gon, as shown in the figure. Let the tswsinase | ele have area A and circumference C, and Jet us say that the area of the polygon mseribed in the enrcle 1s A, and its perimeter is P,, and the area of the polygon ‘which circumscribes to the circle is A, and its The wpe rere" | perimeter is P,, Then A, < A< A, circk 6, Cack ©, circumsaitee the 7 1 1 1 ‘ange Since A, = >-W-B and A, =2-7-R, we get 2-h-B «:) then ht will, get closer and closer to r and the areas A, and of the circle. will both get closer and closer to the area Also, as lt > r we get P, +P, = nr. rrr = nr°, as required. [EZOEES FOS Fine the area ofa cacte with radius 3 cm Solution + 3 em means A m3 = Orem OG the cacumterence of circleis 16x em. Find is area Solution C= 2nr = 16x gves usr So om. m8" = 64x cm" TOS the ares ofa crc 48 em’. Find ts cxcumterence. a6 _ aN _4yBm mR Solution 4 =n1° Areas of Plygorat Regions 106 Wits circte with radius 8 em we draw a square whose vertices lie on the circle. Find the Soluti [ECO 107 t the sure, the ven three ctrcles are Solution area of the region between the square and the cizdle. A square is a regular polygon, so the center of the cixcle and the center of the square are the same point. So we can say that the diagonal of the square 15 equal to the diameter of the curcle: 2 = d = av2,1e.2-8 = a/2and a= 18-8 fem. 2 Mace heute (m8) — v2)" (64 ~ 128) om* tangent (0 each other at the pots A, B and Cand the pomts 0, 0, and 0, on AC are the centers of these circles. The radu of the smallest and middle-sized circles are given as Jem and 3 em respectwely. Find the area of the shaded region. Let the radu of the circles with centers O,, 0, and O, be r,, 7: and r, and let their areas be A,, A, and A, respectively, We know that O,, 0, and O, lie on the line AC. Since AB = 2r, = 2 emand BC = 2r, = 2-3 = 6 em, we have AC = AB + BC = 2+ 6 = 8 cm, S0AC = 2r,= 8m, te. r,= 4m. So the area of the shaded region is, Ay Ay Ay = =n (6-9-1) +x 6rem’, Areas ef Regular Polgenn ara Cites TOS A» cquiaterat sang with side Tength 12 em 1s mserbed im a carcle and arcumsenibes another exele, Find the diference m area of these two cacles, Solution We know that the area of an equilateral triangle is Equating these expressions gves us 4 2 From this we get r = 2V3 em and R = 4¥3 cm. So Svemes teeta = Aviat st ~ mate sin mR? ny? (43) —r. (203)" 48n—12e 36n em. Check Yourself 16 1 Acucle has diameter 14 units. Fmd xts area. 12°45 _ 12412412 aR 12-12-12 aR 2 ‘The eixeumference of a circle is 4 units. Find the area of the circle. 3. A aircle is ansenbed in a square with side length 8 units, Find the area of the region between the square and the circle, 4 Im the figure, ABCD 1s a rectangle whose shorter side measures 10 em. The two circles are internally tangent to this rectangle and externally tangent to each other. Find the area of the shaded region 5. Inthe figure, AABC 1s right mangle and the exrcle ciroumseribes the triangle. Given that the legs of the mange are 6 and 8 units Jong, find the area of the shaded region, Areas of Plygorat Regions The two circles in the figure have centers at B and C, The carcles are anternally tangent to each other at point A. Given that the radius of the circle with center B is 12 em, fd the area of the shaded region. Answers 4 : 149 2 -3.64-16n 4. 200-30m em? 9. 25n-24 6 108Rem* ® C. AREA OF A SECTOR sect A sector of a creular region 18 a repon bounded by two radi of the evrle andthe axe between the endpomts ofthese rada Im the figure, OA and OB axe radit of the corte, isthe angle between the rad, and ‘AB ws the minor are berween the endpomts of these radii. So the shaded area is a sector of the ewcular region Note Inverter. mcAPe< 18m pe, ‘Aand B together with the points on the circle that lie m the exterior of ZAPB form a major are of the circle, denoted by three letters such as ACB. The poms A and B together with the pomts on the cmcle that hee in the interior of ZAPB form a mmor are of the circle, denoted by AB. may ‘The area of a sector is the product of the area of the circular region and the ratio of the angle ‘a between the radi of the sector to a whole angle (360°): aver = 369° | @! Proof Let us divide a whole circle mto 360 congruent slices from its center ‘area of thecircle _ 1 360 © 360° Ifa sector has angle cr between its radii then We need «slices to mateh its area. So the area ‘Then the area of one slice is, er a of the sector is mw ‘350° 360° Areas ef Regular Polrgern ara Ci Ems 109 Solution Ens 110 Solution ENUg111 Solution E112 ‘The shaded region in the figure is a sector of, the circle. Find the azea of this sector. Gen that the angle a. 1s 60° and the radius of the circle as 8 cm, we have k a. 60 ‘awe ~ 360" 360 In the figure, AOB 1s a sector. Given that the Jength of the mmor are AB 1s 4% cam, find the area of the sector. ont We know that the length of an are is 7. So which means 90.=720°, «= 80°.Se Aone, = —oonrt = 80, 360°" ~ 360 2.9% lan em? In the figure, ABCD is a square and A is the center of the quarter crcle ABD. Given that one side of the square measures 10 em, fd the area of the shaded region, We can see that one side of the square is the radius of the aarcle, Sor = 10 cm and m(ZDAB) = 0". So Aanteamia = Argan ~ Ares = 10" ew %. 10°)= (100 ~ 25m) em’, In the figure, ABCD 1s a rectangle and the sector is part of a circle with center at point © which 1s tangent to the rectangle at pont E. Given that AD areas of the shaded regions. 4om, find the sum of the Aros of Plygora Region Sector AEB 1s a semicircle and m(ZAOB) = a = 180°. Smee AD = OF = r = 4 em, we have AB = 2r = 8 em. So A, Aasstrsse = Ancoade ~ Arcaunie 32 — Br) em’, [EQUI 1 1B A crate win radius 6 emis given. AB isa diameter of this civele and BC isa chord. Given that mm(ZABC) = 45°, find the area of the region bounded by AB, BC and the minor are AC. Solution Let us draw the figure and let O be the center of the circle. Drawing OC pves us 0B = 0c =r. Smee m(ZOBC) = 49° and AOBC 1s asosceles, m(ZOCB) = 45° and m(ZBOC) 90", fcegmert wiich cennects | AOC 18 a sector and AOBC 3s a night triangle, so ‘wo points on a crcl, 90 gt 8 = Sones A, Aco + AQOBO) = 5 A chord isa eraight Une (EQNS 114.4 man nas a rectangular garden with sides of length 4m and 6 m surrounded bya fence. The man attaches a sheep to a ten-meter rope which he fixes to the middle of a longer side of the garden, What 1s the {otal area of grass outside the garden that the sheep can graze? Solution Let us draw an appropmiate figure (above night). In the figure, A, A, Ay A, and A, are the A, and, quarter circles that the sheep can graze. As we can see m the figure, A, A,= A, So 4, = 2 10" =29n 360 toa ara = 24, 124, 124,=2 2901 2-89 58 = 30n + 20m = 79 m' EXAMPLE In the figure, ABCD is a square wath side Exmg115 G sa i , Jength 12 cm and A and C are the centers of, two quarter circles. Find the area of the shaded region i Solution Let us draw the diagonal BD. We can easily D conclude that A, = Acand 1» Also, Ay Ay = Avan — Anemtco 90 aot 2 1 Gay el) (144367) em* A(AABD) ~ A, G0 A.rsssmgn = 2A2 = 2: (B6n—72) = (72m 144) em’ 116 tthe sare, aAsc isa right tangle with m(ZBAC) = 90", and 0, 0, and O, are the centers of the three semierees, Given that the area ofthe semicircle with center at 0, is 20n cm’, find the sum of the areas of the semicircles with centers at 0, and 0. Solution Let the radii of the semicizcles centered at 0,, 0, and 0, be ¢, b and a respectively. ‘Then AB = 2c, AC = 2b and BC = 2a By the Pythagorean Theorem we have (2c)" + (2b)" = (2ay', ie. +" = a Given that the area of the semicircle with center at pomt O, 1s 20%, we have rat 5 = 90K, 10. 4 2208 The area of the semtencles with centers at 0, and 0, are = ana 0 etn wt respectively So the sum of thetr areas 1s == 2° 2 Aros of Plygora Region eee ee angle whore vertex les a the center ef a circ, Note More generally, t can be shown that the sum of the areas of the circles whose diameters are the legs of a right triangle is equal to the area of the eitcle whose diameter is the hypotenuse. Check Yourself 17 1. Assector of a circle with radius 12 unitsis given. The central angle of the sector is 120°. Find the area of this sector 2. Amare of circle measures 24 cm. Given that the radius of the cucle as 8 cm, find the area of the sector created by this arc, 5. In the figure, AB isa diameter of the curcle. Given that AB = 16 and m(ZABC) = 30°, find the area of the shaded region. ‘The figure shows a quarter circle with center at point A. Given that AC = 8 and BD = 3, find the area of the shaded region, In the figure, ABCD is a square and pomnts A, B. C and D are the centers of four congruent and tangent quarter circles. Given that AB find the area of the shaded region. 6. Im the figure, A and D are the centers of two tangent quarter circles and ABCD is a rectangle with shorter side length 6 units. BC 1s the diameter of the semicircle BFC. Find the area of the shaded region. Areas ef Regular Polygons ara Ciro 4 Dae In the figure, ABCD 1s a square with side length 12 units, and B and D are the centers of two tangent quarter circles. Find the sum of the areas of the shaded regions, Answers 1 48x 206m" 9 BE +16d8 4 162-20 9.369% 0.72 7.44 M4dn + Tam D. AREA OF A SEGMENT Proof segment A segment is a part ofa circular regon which 1s bounded by a chord AB and the arc AB In the figure, AB sa chord and AB i an are The shaded area shows the segment of the circle that they form area of g segment The area of a segment created by a chord AB 1s the difference of the areas of the sector including the are AB and the triangle whose vertices are A, B and the center of the exrcle: a de Acoma = ggeP* ~ GT sin. Look the igure. A, + Ayis the area ofthe sector whichis bounded by the minor are 4B. Aso, A, am atea of am isosceles triangle with sides rand vertex ange So the area of the segment 1s A, + A,~A,, which 1s the difference of the area of the sector and the area of the isosceles triang) Aros of Plygora Region TV 7 inthe se, 0 is the center ofa circle with radius 6 ei, Given that m(ZOAB) = 30°, find the area of the segment bounded by the chord AB and the minor are 4B. Solution Let us draw OB. Then (04 = 08 = 6 mand m(ZOBA) = m(ZOAB) = 80" which means m(ZAOB) = 120750 Ame =ligar? He sina= On 6) = 360° 2 ‘360 6° sin 120°) = (12 - 973) cm’. [EXOIIE 1:18 te neure shows o cree wth radnis 8 em Given that m(ZACB) = 30°, fd the area of the shaded region. Solution First ofall let us find the central angle for the aro AB. ZACB is an inscribed angle and m{ZACB) = 30°, so ‘m(ZAOB) = a = 2-30" = 60°. So ie prana. ‘xn angle ABC formedby thiee pounte A, 8 and © 60 on de on the earumierence of geet 8°)—(5--8° sin 60") a cirele ic called an 2 sncenbed angie 163) em* 119 4 circ cxreumseries a square with side Iength 8 em, Find the area of the segment created by one side of the square and its correspondmg minor are Areas ef Regular Polgenn ara Cites Solution 1 Solution 2 The diagonal of the square is equal to the diameter of the circle. So 2r = 8V2, 1€ 1 = 42 em, Since the sides ofthe square are the same length, the areas ofthe four segments ae all equal. So (ay2)* - Aansetsnee 1 = 4V2 cm (from above) and the central angle “BOC measures 90°. So Agee = OO nan" -h (4NB)* +sin90” =(8 n=16) om 120 tHe agure snows a eaete with center 0 and E Solution radius 6 em, Given that AB || DC and ‘m(ZBAC) = 30°, find the area of the shaded region. ‘m(ZBAC) = 30° so the measure of the central angle of are BC is m(BC) = 2- m(ZBAC) = 2 - 30" = 60" and m (Ab) = m(BC) = 2. 30° = 60°, so mbt) = 60°. So = 2+ 948) emn*, Check Yourself 18 1 Accircle has radius 16 cm. Find the area of a segment of the circle if its central angle measures 30° 2 The figure shows a sermeucle with diameter AB. Given that m(ZABC) = 45° and BC = 6¥2, find the area of the shaded region. 4 6 B 3. A regular hexagon is insenbed in a ele, Given that one side of the hexagon measures 12 units, find the area of the segment bounded by a side of the hexagon and its corresponding minor are. Areas of Polygon Regione 4 Inthe figure, ABC 1s an isosceles night tangle 4 and BC 1s the diameter of the semueucle, Given that AB = 8, find the sum of the areas of the shaded regons. 8 Ni — CN 64 . ea 1. 5-64) em 2. 9n-18 3. 24m ~ 36V3 4.16 E. AREA OF A RING cee¢e Concenisie cislee sre circles which bhove 3) common mung The area between two concentnic eireles is called a ring. In the figure, O 1s the common center of the careles and r and R are the respective radi of the smaller and bigger cxreles. So the shaded regon is a ning. ‘We can easily see that the area of a rng 1s the difference of the areas of the larger and smaller circles: 1 Ag = ARE 121 tHe eure shows two concentric eucles with common center at A. AB = 3 and BC = 5 are gven, Find the area of the shaded region. Solution The shaded region as a zing. We are given AB = r = 3 and AC =R= S0 Avg (eB) 3") = Gin On roar ef Regular Blygerm ara Ciro 122 the scure shows two concentric circles with center 0. The difference between their curcumferences 1s 8x cm and the area of the shaded region is 487 cm’. Find the radius of each circle Solution The difference of te eucumferences of the axcles 1s 2nf — nr = 8x, which ves us R—r=4, Also, the shaded area is xR —nr* = 48m, so R—1" = 48 Wrong R° Rer=1. Solving R — EZUIES 123 te neue shows wo concemune cuctes. AB 1s a chord of the larger circle and 1s tangent to the smaller circle at point C, Given that AB = 12 cm, find the area of the shaded region. = (R + 1)- R=) = 48 and using R—r = 4 gwes.us (R +1) -4 = 48,10 4 and R + r= 12 simultaneously gives us R = 8 emand r= 4em. Solution Let the sadius of the larger circle be OB = R, a and the radius of the smaller cirele be OC = + Let us draw OC. By the properties of chords we have OC | AB and AC = BC = 6 em. So OB = OC + BC’ ie. BC? = OB OC? = RP -r° = 6° = 36 So the area of the shaded region is rR’ — rr" = (Rt [EXOT 1.24 the nee shows two concentric cites centered at point O. A sector AOB with a 60° central angle is drawn in these circles. Given that OC = 3 em and CA = 3 om, find the area of the shaded region, Solution We have a = 60°, r = 3 comand R= 6 cm. Combiming the formulas for the area of a xmg and the area of a sector gives us the answer: _ 60 = Sgn Aas = pec (ait ~ nr 60 Aros of Plygora Region Check Yourself 19 1. Two concentnte circles have rad 12 cm and 9 cm, Find the area of the region between these two circles. 2 The figure shows two concentric semicircles with common center at the point O. Given that AB = 2 and AD = 12, find the area of the shaded region. Ao8 c &) Ss 163nem? 210m 3. rem" cy F, RATIOS IN CIRCLES ‘We can easily sce that all circles are similar to each other, Thetr ratio of similanty is the same 3. The figure shows three concentric carcles centered at pomt O. The radu of the circles are 3 em, 4 em and 5 cm, m(ZDOC) = 80° and AC and BD are diameters of the biggest circle, Find the sum of the areas of the shaded regions. Answers as the ratio of their radii Let rand R be the radu of two eucles. Then 4. the rauo of sunray 1s = 7 2. the ratio of their circumferences 1s ratio of their areas is If =[ 2 ee tt (2) Furthermore, if AOB and COD are sectors of, ‘wo concentnie circles as shown in the figure then gp, ACOD)_ (rye r 7 R A(AOB) R 4a I) Areas ef Regular Polygons ara Ciro 125 the circumference ofa circle is Sr-em and the ratio ofthe area ofthis eile tothe area of a second circle is >. What is the radius ofthe second circle? Let the first circle have radius v and area 4,, and let the second circle have radius f and area A, ‘The cucumference of the first circle us 8 = 2nr,Le.r = 4 em, Pom Proper 128 we ow ta So Ady a(R 8m [EDOIE 126 20 ture shows a sector oftwo concentric Solution ENE 127 ‘carcles centered at pomt 0. Given that a AC = 3 cm, OC = 6 cm and ¥ S, = lom om, fd the area S, We have r = OC = 6 emand R= 0A=9 em, By Property 12.3, (CD) Ss A(OAB) 545, 5s, Using S, = 15 ges —_S: s " 1om+5, =4 and by cross muluplying we get 98; = 60m + 4S., 12. S.= 12cm’ A pizza shop sells three types of pizza with diameters 12, em, 24 em and 36 em. The price of the pizza with diameter 24 ems $4. If all the pizzas are priced in proportion to their areas, what are the prices of the other pizzas? ‘The pnices are related to the sizes of the pizzas. ‘The area of the small pizza is. 6° = 36n, the area of the medium pizza ts n- 12° = 144 and the area of the large pizza is m- 18° = 394m. Areas of Plygorat Regions Now let P,, P, and P, be the prices of the small, medium and large pizzas, respectively. ¢, 36. and so P, = $1. Similarly, van 4 Check Yourself 20 1. The ratio of the cixcumferences of two encles 1s 3. Given that the area of the smaller curcle is 12%, find the area of the bigger circle. 2. The figure shows sectors of two concentnic circles ©, centered at point O. Given that OA = 9 em, AB = 3.cm and the area of sector OAD is 36x em", find the area of the shaded region 3. We want to increase the area of a citele by a oo a3 8 factor of 3. By what factor should we increase its radius? Answers 1.108% 2. 28m em* 3.43 roar ef Regular Blygerm ara Ciro SOLVED PROBLEMS "LIDEE \ sic) Anctent geometers studied many mathematical problems, but three BP] prootems decame particulary famous in the ancient world. They BY) were famous because nobody was able to solve them using the r traditional tools of compass and straightedge (1.e. a ruler with no marked measurements), These problems were — 1, squaring the circle . 2. trisecting an angle, and BO) 3. doubling the cube. These problems were studied by some of history's most famous geometers, and mathematicrans stil talk about them today Problem 1: Squaring This problem challenged geometers to construct a square that has exaclly the same area as a given exrcle, using only a compass and straightedge. "| We know that the area ofa circle 1s A = ay" and the area of a square 3s A Therefore the problem is the same as ff a°, which we can finding the solution to the equation 7y* rewnite as, Hindu mathematicians tried to solve the geometric problem. They learned how to convert rectangles to squares with the same area, but they could not construct a square from a curcle. After this, Greek mathematicians worked on the problem bur they still couldn't solve it. However, algebraically we can sce that because ris an irrational number and © ssa rational number, squaring the circle precisely is impossible Problem 2; Trisect an Angle BF) Diviaing an angle into to equal parts (called bisecting the angle) is Han easy construction with a compass and straightedge Mathematicians also thought that it would be easy to divide an angle Inco three parts (called triseeting 1) with the same tools, but fact BD they coutantt do xt. Archimedes and Pappus both showed how to BY) eescccan angie with a marked ruler but they could not do it with an unmarked straightedge. Today 3t 38 known that there 2s no general method for trisecting an angle by construction, Probl: 3, Doubling This problem asks geometers to construct a cube that has exactly twice the volume of a given cube by using only a compass and straightedge. The Greek mathematician Archytas studied this problem, but he was unable to solve 1 [ vith only a compass and straightedge The volume of a cube with side a is a°. Doubling this cube is creates a second cube with volume 2a’. One side of this cube must measure 32. Since this length cannot be constructed directly with a compass and straightedge, the problem 1s. fim impossible to salve. , pronounced Like ‘pie’ an English, is a special number an geometry that Links the diameter and circumference of a circle, To nts first few decimal places, w = 3.1419, although the decimal expansion of r continues infimitely mis an wrational number, which means that it cannot be written as the ratio of two whole numbers, Throughout the history of math, people have used rational approximations to such as 3 (used by the Babylonians), and a number fh ‘ between 2 and 3° (used by Archimedes). Other ancient W mathemancians Used the fraction =, which 1s approximately equal to x with a difference of 0.04%. The ‘mathematician Euler first used the famabar r symbol for the . number in 1737 (Over time, mathematicians have found more and more digits in the decimal expansion of x. Using today’s moder computers, mullions of digits have been calculated. There is even a special day im honour of this number: since ris. approximately equal to 3.14, March 14 of every year (the fourteenth day of the third month) is named Pi Day. ~ t 3¢ in the d 1 sion oi ¢ m = 8.1415926535 8979323846 2643383279 5028841971 6939937510 5820974944 5923078164 0628620899 8628034825 3421170679 8214808631 3282306647 0938446095 ANNIE 5058223172 5359408128 4811174502 8410270193 8521105559 6446229489 5493038196 4428810975 6659334461 2847564823 3786783165 2712019091 4564856692 3460348610 4543266482 1339360726 0249141273... A. Area of a Regular Polygon 1. Acircle mscnbed m a square has radius 6 units. Find the area of the square 2. The radius of the circumsenbed crcle of an ‘equilateral triangle is 4 units, Find the area of the ‘equilateral triangle. 3. One side of a regular hexagon 1s 10 units long. Find the area of this hexagon. 4, The rads of the circumsenbed eucle of a regular octagon is 4 units, Find the area of the octagon 5. Im the figure, ABCDEF 4 b sa regular hexagon, Guven that A(ABCDEF) = 24.3, find the area of the shaded region 6 Inthe figure, ABCD isp c a square and ACDE is an equilateral tnangle. Given that DE = 8, find the area E of the shaded region, 7. A regular pentagon has perumeter 70 units. Given that the radius of the inscribed circle of this, pentagon is 9.6 units find the area of the pentagon. 8. A square is inscribed in a circle. Given that the area of the square is 8 square units, find the area of the regular hexagon that can be mscribed inthis, circle, 9 In the figure, ABCDEF 4 » 4s a regular hexagon 7 with side length 4 units. Given that Gand c H ate the mdpoints of sides BC and CD respectively, find the si area of the shaded region. 10, A regular hexagon has perimeter x cm and area x cm, Find the length of one side of the hexagon. IL, In the figure, ABCDEFGH 1s a regular octagon, Given that BF = 12, find the area of ABP. 12. In the figure, £ ABCDEFGH 1s a regular octagon and 0 a as its center. Given that AD = 12, find the area of the octagon. a 8 18. A circle has radius 6 units, We draw a regular Polygon around this circle. Given that one side of the polygon is 4¥3 units long, find the number of sides of this poly gon. Areas of Plygorat Regions 14, A circle with radius 6 umts 1s mscnbed im a regular hexagon and cucumsenbed by another regular hexagon. What 3s the atea of the regon between these two hexagons? 15, Prove that if two regular polygons have equal perimeters then the polygon with the most sides ‘has the largest area B, Area of a Circular Region 1G, A exle has radius 7 umts. Fund ats area. U7, A cnrele has area 1 square umst. Find its radius. 18. A cucle bas circumference 207, Fund its area 19, Find the area ofa circle whose diameter is 5.2 units. 20. A cucle has circumference 10, Find its area. 21, In the figure, ABCD 1s a rectangle and the two circles are tangent to the sides of the rectangle. Given that the carcles are tangent to each other and both have radius 6 units, find the sum of the areas of the shaded regons. Areas ef Regular Polygons ara Ciro 22, 23, 24. 26. o 28. o ‘Two circles are internally tangent to each other. The distance between their centers 25 12 uns and the diameter of the larger circle is 34 uit. Fund the difference beween the areas of these two carcles. The azea of a carcle is twice the sum of the areas of three circles with radii 3 em, 4 cm and 6 em, Find the radius of Uus circle, A man is connected to a boat with a rope which is, 10 m long. The water is 6 m deep. What 3s the area of the region that the man can walk in? Im the figure, ABCD 1s a rectangle and the three carcles are tangent to each. other. The two small carcles are also congruent to each other, Given that the radius of the largest circle ts 12 units, find the sum of the areas of the shaded regions. From one point on a circle we draw two chords vith lengths 14 units and 48 units. The length of te line segment that connects the midpomts of these chords is 25 units, Find the area of the carcte. C. Area of a Sector 27. A-cicle has radius 12 umuts. Find the area of the sector of this circle which is bounded by an are with central angle 120°. . A circle has radius 6 units. Find the area of the sector of this circle which 1s bounded. by an are with a central angle of 1.5 radians. 29. The figure shows, three central angles with measures 36", 24° and 30°, Given that te radius of the cele 1s 8 unt, find the sum of the areas of the shaded regions. 80, The figure shows a circle and four semicircles, The radius of the larger circle is 8 cm. Find the sum of the areas of the shaded regions. SI. In the figure, SABC is an equilateral mange with side length 8. Given that the mee congruent curcles have centers at ponts A, B and C, find the area of the shaded region. 32, The figure shows a quarter circle centered at 0 yath radius 12 units. The eirde with center at pomt C35 tangent t0 OA, OB and the are AB. Find the sum of the areas of the shaded regons. D. Area of a Segment 33. A regular octagon 1s msenbed in a circle with radius 8 umits, Find the area of the segment bounded by one side of the octagon and the are corresponding to this side. 84, An equilateral triangle ABC has side length 8 units, Vertex A is at the center of a circle and vertices B and C are on the circle. Find the area of the segment which 1s between side BC and the manor are BC. 36, In the figure, ABCD is a square wath side length 6. Given that O As the center of the semicircle and AO = OB, find the sum of the areas of the shaded regions. 36, In the figure, Os the ccenter of the given cnrele. Given that ‘m(ZABC) = 19° and the radius of the circle 18 12 units, find the area of the shaded region, 37, In the figure, 0 1s the center of a carcle with radius 8 units. Given that the measures of ZAOH, 2BOC, ZDOE and ZFOG are all 30°, find the sum of the areas of the shaded regions, Arcus of Polygon Reon 38. In the figure, ABCD 1s a 1) c 43, A regular hexagon yath side length 8 umts 1s square with side length given, Find the area of the region between ats 12 umits. Given that AB inscribed and circumsenbed circles. and BC are the diameters of the two semicircles, find the area of the shaded region. ; 44, In the figure, O 1s the center of both of the ven circles. Given that OD = 9, DB = Sand m(ZAOB) = 60°, find the area of the shaded region. 39. In the figure, ABCD 1s a square with side length 8 units, Given that B is the center of the quarter circle and AB and BC are the diameters of the wo semicucles, find the sum of the areas of the shaded regions. F. Ratios in Circles 45, By what factor should we increase the radius of a E. Area of a Ring circle in order to mcrease its area five times? 40. Two concentric exrcles have radu 3 cm and 9 cm. Find the area of the region between these two oneles. 4G, Two circles with radi r and 4r are given. Find the 41. In the figure, O 1s the ratio of their areas, commen center of two circles with radu R and r. Gwen that R + r= 10 and R—1=m, find the area of the shaded region in terms AT. The figure shows 4 ‘two sectors with % ofm. common center at pomt 0, 42, Two concentne circles have radu R and r, Given. OD = 6, that the sum of the circumferences of these BD = 2and ore 5 ‘0 circles is 18 and the area of the region between. S, = Trare gwen, them is 9x, find the radu of these circles Find the area Sy Areas ef Regular Polgenn ara Cites CHAPTER SUMMARY 1. The Concept of Area + The union of a polygon and its interior region is called a polygonal gon, +The mtenor region of a squame woth sde length one tit ss called a square wnt +The ama of a plane figure is the total number of non-overlapping square units and part units that cover the surfice ofthe figure The area of figure is always positive zeal number An alutude 1s a line segment between a verex of a polygon and a line containinga side ofthe polygon which 1s perpendicular to this Ime, The length ofan alutude is called a height ofthe polygon + 1£ «wo figums are congruent then their areas are the The area of rectangle isthe product ofthe lengths of two consecutive sides: Away = 2b te we connect a pant on p & é lone side of a rectangle to the endpoints of the ‘opposte side then the area of the mangle obsxined 1s balf the atea of the 4 < 2 rectangle 2. Area of a Tangle © Base-height formula: The area of a tangle is half of the product of one base and the height of the mangle from thatbase ‘© Since we can use any side of a triangle as a base, ‘Area of night tangle. The area ofa night triangle is half the product of ts legs ‘rea of an equilateral triangle The area of an equilateral ‘mange with side length @ 1s one-fourth of the product of @and B Heron's Formula: If a mangle has ses a, b and ¢ and ‘perimeter 2u then the area of the triangle is the square oot ofthe product ofu,u—a, ub andu ‘Tgonomeme formula fr the area ofa mangle The area, ofa triangle 1s half the product of any two sides and the sine of the angle between these two dec Equivalently, dabsne = LacsinB = Abesind z 2 z ‘Area of a tangle by fis tnradius: The area of a mangle is, the product of half its perimeter and its inradius, ie. if then Area of @ tang by fs croummdius The area of a tmangle is the ratio of the product of the sidas to four times its eiroumradius: Aros of Polygon Resor atbte 3, Properties of the Area of a Triangle #1 Imanght mange, the 4 oduct of the fags 16 equal tothe product of y the hypotenuse and the Jength of the altitude tog H the hypotenuse —-_— 2 Ifthe base lengths and heights of two triangles are the same then their areas are equal 2 A median of a mangle divides 4 the area of the triangle into two equal pasts, 4. I the heights of two tangles ae the same then the rauoof A their area isthe same as the ato of their base Jonas ia ‘Aga result, we can waite A(AABD) = mS and 5 If the bases of two wiangles 4 are the same then the ratio of Dd thew areas is che same as the ratio of their heights (Chapter Summary 8 The ratio of the areas of the = 4 two tangles formed by the bicectorofan angle ma mangle ts the same as the mitio of the lengias of the two aides separated by the bisector 7 The medians of a triangle together divide the triangle nto six equal parts 8 IFABC is a tangle with inmadius 7 and heights, thy and fi then © Tetwo tnanglas are similar then the rato of their aneasis| equal to the square of the ratio of similarity: cence congener equal parts: Vy t naar assy -aaoeo, fo \’/\. A(ADER) Ifwe divide two sides ofa mangle A ‘mto equal lengths and connect the ses by lines, the areas of the arts are proportional to the numbers S, 25, 3S, 75, a cg 4, Area of a Quadrilateral + We can find the area of a polygon Dy considering the ameas of the tuiangles formed by diagonals from a cingle vertex 2 + The atea of convex polygon c is equal to half the product cf toe lenghs of diagonals 9, ‘and the sme of the angle between the diagonals. a © Ifthe dlagonals of a quadrilateral are perpendicular to each other taen scape) = ABP © The diagonals of a > c quadrilateral diade the sine ZN cr i > Syand A(AADID = 8, then 33285 5. Area of a Parallelogram + Meaiacte png ° isthe product ofthe ont ofany base and the Jength of the caneponding altace i oF é Hae =a, * IEABCD isa parallelogram no © with sides a and b separated by an angle A then AABCD) = @-b- sin A. f n 7 2 » IfPisany » c paint mside a parallelogram AECD then a % ‘AAPAB) +A¢APCD) = aPEC)+ AcaPDA) - MARE) 19s of te Area of @ Purallelogram 4 Any diagonal in a parallelogram divides it into (wo equal areas ae parallelogam diviée 1s 2 r area into four equal ‘arts Taree ines dram fiom any vertex of a parallelogram to the opposite vertex and the ie midpoints of the two non-adacent sides vide the paralelogram into four equal parts © Any pomt E on any side pe ¢ of a parallelogram which is connected to the two non-adjacent vertices crates 2 couangle which has the half the area of the parallelogram AABCD) ACAABE 2 Areas of Polygon Regione Connecting the rmidpoints of opposite sade ofa paralblogram creates four congruent pamullelograms. 6, Area of a Rhombus © The area of a rhombus is ren by MABCD) = base x height — of, A(ABCD) = ah, : my a a zB * Fora thombus ABCD, A(ABCD: sin A The atea ofa shombusis halfthe product ofits diagonals Ac -BD 2 ACABCD) 7. Area of a Square © Theawacfasqumistee =) yc ‘square ofits side length AMABCD) = a-a,18 A(ABCD) = a * A.square shames all the properties of a parallelogram. © The dingonals of a cquare awe perpendicular co each coher Ife is the length of either diagonal of a square then cane) - 40-BD_ € (Chapter Summary 8. Area of a Trapezoid + The area of a wapezoid is the product of the height sn half the som of the tans + In the figure, ABCD isa apeaaid and O is the intersection point of its agonals, IEA(AAOD) A(AKOB) =m, A(ABOC) = y and A(ACOD) ven IEABCD isa trapezoid and E 1s the midpoint of Jeg AD then ‘A(ABCD) ‘manec) = AAR 9. Area of a Kite © The diagonals of a late ax perpendicular to each other © The ama ofa kate is half the ‘product of the lengths ofits ‘diagonals dy 10, Area of «Regula Polygon ‘any regular polygon can be cram made acre so that all ts vertices lie on the etek This excle is called the ctroumeeribed circle ofthe polygon ‘+ Accucle which is tangent to all sides of polygon is called the ansenbed qucle of the polygon + The distance between the center of a regular polygon and cone ofits sides is caled the apothem of the polygon + Ina gular polygon, the centers of the circumscribed ‘and inscribed circles comeide at the same point. This ‘points called the center of the zegular polygon + The distance betneen the center of a regular polygon and any of is vertices iste radius ofits czoumscnibed exe ‘The apothem of a mgular polygom is equal to the radius of ats insenibed circle. + We use r to denote the radius of an inscnbed ale, and Ato denote the radius of cxrcumsenbed circle + The ama of a regular polygon is half the product of us apothem and perimeter Alternatively, A = w-r, where u is half the perimeter of the polygon The triangles that are created when we connect the ‘center ofa polygon to its vertices are all congruent. Their concralangl is = 200 + fava ofa gular potgon = = Se 11. Area of a Circular Region + Tue union of cle and x inenor region i called a ssreuar pon Ta aa or cc aga a mee A square ofthe mdi of cle 12, Area of a Sector # A sector of a circular region is a region bounded by two radii ofthe circle and the are between the endpoints of these radi © The area of 2 sector i the product of the area of the ‘re and the ratio ofthe angle a between the radu of the sector toa whole ange (360°) 18, Area of a Segment + Asogmont isa patt of 1 ciroular region which, “ is bounded by a chord AB and the arc AB, a Seynuent t © The area of a segment created by a chord AB is the fference of the areas of the sector including the are AB. and the triangle whose vertices are 4, B and the center of the trian gle 14. Area of a Ring ‘+ The area between two concentric circles is called a ring © The ama ofa ming is the ownce of the ameas ofthe larger and smaller circles: Aros of Polygon Resor 15, Ratios in Circles ‘All exxoles ate similar to each other and their rate of similanty is the same as the ratio of ches racht ‘© Ifrand R are the radii of wo circles then | the ratio of similarity is w= 1 R © the mati of their excumsezences is the ratio of thew amas is In the figup, if Os the center of the ° two condenmic cicles then @_+ Lr c e aR acoD) ADB) (Chapter Summary Centos 1 What is a polygonal region? How can we define the concept of area? Define the general concept of base of a polygon, 4 Whats the zelauon between the area of a rectangle and the area ofa triangle? > How can we calculate the area of a tangle from Als uiree side lengths? © How can we calculate the area of a triangle if we now any two side lengths and any two angles? How can we find the insadius and eircumadius of 1 triangle if we know its side lengths? How can we find the heights of a mangle if we ‘know its side lengths? © If we know the height of an equilateral triangle, ‘how can we find 1ts area? If two triangles are similar, what ean we say about their areas? 11 How ean the properties of the area of a triangle help us? 12, We can say that the triangle is the most important figure in the study of polygonal regions. Why? 15 Ifthe diagonals of a polygon are perpendicular to each other, we can find its area without using the sine of the angle between the diagonals. Why don't we need to use the sme? etree 14, Why do we use the same formula to calculate the area of parallelograms, thombi, squares? rectangles and to We know the lengths of the parallelogram and one of its interior angles. Can We find the lengths of the alutudes of this parallelogram? sides of a ‘What 1s the relation between the diagonals of a parallelogram and is area? 17 How can we find the area of a square if we know only the length of its diagonal? 28 [have got a cylindrical pencil. How can I find the area of the cross section of this pencil if | know its width? 10 The lengths of the diagonals of a quadrilateral are ven, Can we find the area of the quadniateral if ws ‘. @ parallelogram? b. athombus? 20 If we crease the sides of a square by x%, by what percentage will the area of the square increase? If we know the area of a rhombus and one mterior angle, how can we find its side length? How can we find the area ofa right trapezoid? We know the lengths of the lower base and the legs of an isosceles trapezoid, How can we find ats area? 24 We know one of the diagonals of an isosceles ‘tapezoxd, Given that ts diagonals are perpendicular to each othex how can we find its area? 28 ‘What 1s the apothem? How is 1t useful? If the centers of the inseribed and circumscribed carcles of a polygon comerde, what can you say about this polygon? How can we find the area of a regular polygon? How can we find the area of a polygon which is not regular? How can we find the area of a sector? How can we find the area of a zing? How can we find the area of a semicircle? ‘Aring is given. We cut a part of the ring which has central ange measurmg x°, How can we find the area of Unis part? Can we compare two circles? How? Can we find the area of a circle if we know its circumference? ‘Two regular polygons with 10 and 12 sides respectively have equal perimeter. Which of them has the biggest area? Why? Acarcle mscribed in a square has area A. Another square is Inseribed in the eirele. Find a formula in terms of A for the areas of the squares. Aros of Plygora Region 1. In the figure, ABCD. c asa tectange and m(ZBEC) = 30°, a Given that AE = 2and EC=6,fndthe 4 > * ® area of the rectangle Ab +98 B)® c)12 Db) 9+ 63 E) 12+ 43 % Ina miangle AABC, a = 12 cm, b= 8 om and h, = 6 em. Find the length of A)4em 8) 6em C)8em D)9em ©) 12cm 3. Im the figure, ABC 1s 4 a night mangle with m(ZC) = 90" Given that 3 lb m(ZA) = 45°, AB = 12 and 4 De = 49, fmd the al area of ABD. - 2 Ay6 B)8 C122 Dj)12 BB 4, ABCD 1s a rectangle and ABE 1s an equilateral tangle such that £ hes on CD. The height of the equilateral mangle 1s 12. units. Find the area of the region between the equilateral mangle and the rectangle. A)24-B)36 C24) 48 CE) 483. (Chapter Raviow Tt £ A tangle has sides of length 18, 12 and 10 unuts. Find the height tots shortest side. D) 12 FE) BE As B12 C6 6. In the figure, AB = 12, CD = 6 and ‘m(ZABD) = 30°. What is the area of aacD? A12 B14 O18 D4 HR 7. Inthe figure, ABLBC 4 and BD 1 DC. Given that AB = BC = 13 and > BD = 5, find the area of ‘quadrilateral ABCD. A)39 B32) 36D) 30) 4S, 8. The legs of a nght mange are 6 and 8 units long. Find the sum of the diameters of the carcumsenbed and mseribed circles of this mangle AT Bs O10 D4 HIB 9. The hypotenuse of a night tangle measures 10 units. Given that the radius of ats insenibed ele 15 2 umits, find the azea of this mange. A)24 BAB C12 DBE) IS UO, The sides of a tangle are a = b = Sand c = 6. Find the length of the diameter of te circumscribed circle of this triangle jim mo B pes wis IL. In the figure, 4 AB = 6, | EC = 3 and BD = DC = 9. A what is A@*BC) » A@EDC) =f 5 7 26 a6 Bb oO 2 py 6s Hh 6 B) > FD 1s 12, m the figure, BD and CE are medians of the tangle, Given that A(AABC) =120, ‘find A(ADEG). c Aj40-B)20° «Cy 12 DO EVD 13, A cxcle mscnbed ma night tangle divides the hypotenuse anto two parts with lengths 4 and 6 ‘umits at the pomt of tangeney. Whatas the area of thus triangle? 24 B12 C48 D386) U4, The Legs of a night tmange are 12 and 16 units Jong. Find the length of the alttude drawn to ats hypotenuse. Ao B96 ©)20 D190 £48 18. In the figure, BC is A divided into four equal parts and AF is divided into three equal parts. Given that A(AABG) = 8, find A(AABC), Ay24 B32) 34D) 3B) AD. 16, In the Soure . DH LBC. HAD = 2-e, ‘BC = 12 and DH = 3, whats the area of ABC? a i t A386 B54 C18) BS Areas of Plygorn Region I, The sides of a quadrilateral ABCD are given as AB = 24, BC = 20, CD = 19 and AD = 7, Given the diagonal BD = 25, find the area of ABCD. A) 125 B) 254 ©) 300) 168) 234 2. Im the figure, ABCD 1s ¢ a quadniateral Given that A(AAOB) = 8, Ds A@aDOC) = 12 ang ASAD) _ 2, ¥ s A@BOC) 3 fund A(ABCD). A)36 -B)40 C38) 4a) 8 3 Inthe figure, ABCDis.——)_ c a parallelogram, ee eva tit zoe) = fae ® ‘m(ZOAB) = a ® AO = 8 and BO = 6, find the area of ABCD. A)96 5) 48/2 C)96V2 D) 60/2 FE) 84 4, In the figure, ABCD is a parallelogram, Given that AE and DE are the bisectors of ZA and 2D respectively, AE = 16 and De = 10, find A(ABCD), A)192 B) 144 C) 126) 108 -E) 96 Chapter Ravi Tat ? 5. In the figure, ABCD 4a parallelogram, Given that AE = EF = 5B, 3. BG =2.DG t cn and A(AEFG) = 6, find A(ABCD). A848) 120 6. In the figure, ABCD 1 a parallelogram, ©) 60 Given that AE 1 EB, AE = Sand De = 13, find A(ABCD), A608) B4 7. In the figure, ABCD isa rhombus and AE and BE are angle Diseetors, Given that AE = 6 and BE A368) 38 8. In the figure, ABCD is a rhombus. Gwen that BE 1 DC, DE = EC and BE = 8 cm, Ond. A(ABCD), A) 128 135 3 Dd) ©) 48 a a ©) 24 2) 64 p90 E45 p65 E72 4, find the area of the shaded region, Di 548 a: ¢ ©) 1985 5 OE 3 9. A square has perimeter 20 em and the ratto of the area of this square to the area of another square as 1. Find the penmeter ofthe second square A) 94m 1B) 40 em ©) 60cm 1p) 48 em ©) 180 em 10. An equilateral tangle has area 12V3 square units. Find the area of the square whose perme teris equal to the perzmeter of this equilateral tn: ange. A)18 B)27 C) 24D) 248) 363 IL. In the Bgure, ABCD is 4 cn a trapezoid and BF 1 its median p= + {5 Given that EF = 10cm, 4 B CD HB and HB = 8 em, find A(ABCD). A) 80 cm" 8) 60 em" ©) 50 em 1D) 40 em* 5) 35 em* 12, In the figure, PQRS is 5 a night trapezoid Given that QR = 4, 5a SR = 6¥2 and m(ZQRS) = 135°, find the area of this trapezoid. 4 oa “0 A)36 B52 C242) 36/2 E) 4d 13, The bases of a trapezoid ABCD are AB = 6 m and DC = 4m, We draw the median EP. Find the ratio of the area of ABFE to the area of EFCD. ot 9B of pt ptt B q 8 14, ABCD is a trapezoid pe and E 1s the midpoint of AB. Given that AB = 4-DC, find A(ABCD) A 7 a AQ@DEC)” 3 B44 O8 DE HT 15, A trapezoid has height 8 units and area 48 unit” ‘The difference between the lengths of its bases 1s 4 units. Find the length of the shorter base. As 87 O68 DS BHA 16, In the figure, ABCD is a 2 Aue, Given that wa AB = AD, BC = DC = 443, m(ZBAD) m(ZBCD) ‘find ABCD) A) 24/3 8) 48/3 0) 64D) 96 F) 728 Areas of Plygorn Region 1. In the figure, ABCDEFGH E is a regular octagon \ centered at O, What are © 7 Motto | B)12:6:0:8 0) 6:4:3:3 D)7:3:4:4 5) 9:3:424 2. ABCDEFGH is a regular cetagon with side length 4 units, Find the area of ABD. By 4 +43 O16 FE) 6+ 80 Aye +40 D) 6 + 82 3. One side of a regular hexagon measures 14 unuts. Find the area of this hexagon. A) 196 B) 2043 D) 204 E) 1498 ©) 1963 4. The radi of the circumscribed and msenbed circles of a regular polygon are 6 and 342 units respectively, Find the area of this polygon. A) 549 8) 36/3, c) 72 Dy 182 £) 96 5. A circle has circumference 18V2n cm. Find its A) 812 em* —B) 36V2mem* ©) 81/2rem* D) 3247 cm" ©) 162n em" (Chapter Rave Tt 3 The figure shows two carcles which are internally tangent to each other at A. O 4s center of the bigger circle and OB = 1, BC = 8. Find the area of the shaded regon. A) 64x 8) 56m C) 81x D) 40m) 17 7. Acircleas msenbed m a regular hexagon with side length 8 units. Find the area of the region between the hexagon and the circle. A) 64-48% 8) 16V3- 16x ©) 96 - 24% D) 963 ~ 48% E) 36x - 2748 8. The two congruent cureles 0, 1m the figure are tangent to each other and to the sides ofthe square ABCD. Given that these circles have radi 6 units, ind the area of the shaded region A) 48/2 + 144-365 ©) 4-728 3) 216¥2 - 36x D) 216 ~ 72 E) 216 + 14492 - 72n 9. In the figure, ABCDEF . o 38 a regular hexagon vyath side length 6 units and C is the center of the sector. Find the area of the shaded region. -o 8 A) 948-128 B) 36m ©) 144-72, ‘D) 216 - 72n ) 1283-1 10, In the figure, 0 3s the nl center of the sector and the circle wath center at Cis tangent to the sides 0 and are of this sector, Given that m(ZAOB) = 60° and the radius of the small B circle is 4 units, find the sum of the areas of the shaded regons. A)3n BB) 4m C) Be iD) Bn) Oe. LL, In the figure, ABCD is a © square with side length >< 12 units and A and B are the centers of the two quarter circles. Find the area of the shaded regon. a a a A) 86x 2) 2a ©) 48m - 36V3 D) 24m - 123 F) 363 + 120 1, In the figure, 01s the center of the ence. Given that m(ZOAB) = 30° and AB = 18, find the area of the shaded segment —_ A) 480 B) 12m + 3V3 ©) 36x D) 48n— 183 E) 36m - 27V3 13, In the figure, ABCD 3s a rectangle and AB is the diameter of the semicircle. The semicircle euts DC at Band F, Gwen that AB = 8 and FC = 2, find the area of the shaded region A) 4 - 243, B) 3n-3V3 ©) on b) Baa 5 Bag 14, One side of a square measures 6 units. Find the area of the region between the inscribed and. circumsenbed circles of this square by 2 3 A) Gr B) 8x C) 9m E) 6n-36 In the figure, O 1s the center of the given sector. Given that m(ZAOB) = 49°, OF = 5, FD = 2and DB = 1, find the sum of the areas of the shaded regions, nie moor van mH BE 1G, In the figure, O 1s the 24 center of the sector. Given that OB = 8, AB = 2and °. S, = 16, find the total area of the sector AOD, > > A) 25m PB) 24m C) 16k D) 18" FE) 20 Areas of Plygorn Region 1. In the figure, AP 1 an angle bisector. Given that PH LAH, PH = 4,4 BP = Sand AH = 7, find A(QABP) AS BT 6/8 2, In ABC, BP and CP are 4 angle bisectors. AB=9, AC = 7and A(AABP) = 20 are given. Find A(AAPC). A)30° 0 B)28C)27, DBE 3, In AABC, KC and BL A are medians. The sum of the shaded 7 areas is !7 square 7m units. Find A(AABC) A9 Bl OM DIT Ha 4, In the figure, G as the n centroid of AABC, Given that AK =3-BK, ‘m(ZKGA) = 120°, AG = 4 and, GK=23,fmdthe area of AABC. AIG B20 C24) 2B_—s«*Y 32. (Chapter Raview Tt ¢ 5.In AABC opposite, poimts D and E are the midpomts of BC and AC respectively X Given that PH DE. » HE = 8, HD = 4and PH = 9, find the area of ABC. a 7 “ Ay24-B)3S 4B) SAE) TO. 6. In AABC opposite, m(ZEDP) = m(ZDEB) = 90°, Given that DF = 3, DE = 4 and BC = 16, »y md A(AABC) Ay32-B)36 40D) AD_— EAB 7. Inthe figure, ABLACand = 4 AD. BC. Given that BL = 6 em, AL = 62cm and LD = 4em,fna 6 the area of ALDC. D A) 24 am" 8) 26 em" ©) 28 cm* 1b) 32 em* ©) 36 cmt 8, The medians of a tnangle are 3, 4 and 9 unuts Jong, Find the area of this tangle, 6 87 O8 Ds #2 9. CABE opposite is a g quadrilateral. Gaven that CAL AB EB = AB,CD =3, DB = 12 and m(ZABC) = m(ZEBC), a find A(AACE), AIG B18 C)24— iD) 28 E36 10, In the figure BA\|CD, A(SABE) = S, A(ABEC) = Sy, A(ACED) = S, and 52 s, a os gis at pt ot mi wt 3 5 5 LL, tm the ngure g AB ||CDIIEP. Given that AB=6, EF = 1 ana Bt é we f sro ge and ane length CD AT BE OF Do HE E) 83 ‘12. In the figure, ABCD is a rhombus. E and F are the midpoints of the gaven Ime segments. Find the area of the shaded region if, A(ABCD) = 9043. AWS B33 C48) 6S 13, The semmewcleim the 4 figure 1s centered at 0. Gwen that B and T are poits of tangency, m(ZACB) = 30° and AB = 4, fnd the area of the shaded region. ax A) 88-5 B-s an i6a- D) 16v8- ©) 183, By syne F) 63-3 14, im the figure opposite, the radius of the circle is r= 2, Find the sum of the shaded areas A) 2m - 33 B) ant 3 ©) 4n- D) an +3v8 5) 3n- 38 15, The two circles m2 € the figure have equal adn 6 umits and are tangent to each other 9," at point A, BC 1s \ tangent to the circles at B and C. Find the area of the shaded region Ay 72 3) 18x36 1p) 3649 Canto ) 72-18 16, ABCD and BEKF opposite are squares and pomt K 1s on the quarter circle centered at B, If the shaded area is 16 square units, find AGBCD), Ay36-8)38 42) 4A GB Arcus of Flyer Reon ANSWERS TO EXERCISES eee 1.296 2192 5 49/3 464 5.24 6 Bando 7.38 5.18492 9. 10.32 11.30 12 P15. 60 16125 17.60 18.8 19.64 20 PE 2, 8» yo 14.1208 10, n, = 18, 3 230.368 b 18 ME a4 a 98 b MME cov 25.39 26.04/98 b 638 27 Ati 98 512 9 IP gs sar 28.2003 29.a, 12 b.12V2 24 d. 123 30. a aT 06 aM 5 or 8 op res 8 Annet 0 4 so oo om uD 4, 26 7 ee aa 180 49.10 46.7 47.5 48.55 49. > 00.40 ol. r= W5,R Tp 82-24 83.2 54.30 v5. 08-108 96. 28 45 57 88 55, MEP oo 9 AGhaded triangle) G5 00, Hint All uhree shaded tnangles are similar to ABC. Find the rates of “CHET E ider theix sum and cross multiply ers 172248 3) 44844080 5.56 6.153 781 af 9.12em 10.4 11.6v3 12.12 13.96 ays : 14 AB 15,18 16 405 17 140 15.18 19,98.8 20.40 21,144 22.42 23.8 24 110em* 25.3E 1285 5 a5, EYE 27, gor 25, aVBand 20,6 28.120 80.12 31.1000 32.196 33.508 54.160 80.45 30.18 87,40 35.1325 39.100 40, 96% 41, 336-1923 42.7 45.32 44. ; 49.18 46.64 47 z 45.89 94,230 99, 192 96,200 97.240 28. 48 40,10 50.244 165 91.22 92.244 98 35, SEPT 99.48 60.64 61.3 62. 96em® 63. 1836 64 36/3 69.24 65.30 67. 18 + 18V3 58.488 Arawore to Exoriver Lad 2126 3 1308 428 0 168 064-168 7.396 6.66 9.86 10 MF om 11 188 1 29 18 100% 19.6.76n 20. > ve © 12,288 - 1144/2 13.6 14.183 10, Hint: Use induction. 16, 7m 1 21,288 72m 22.264n 23.10cm 24. 64mm” 25. 432 + 288V2— 216m 26. 625m 27. 48n 28.27 29. 16K 32m 45m 30. 32m em* 3). 16V3-8r 52 Q88V9—206)r 99. 8n— 16/2 34. —*-16Y3 39, 736 36, 6On- 36 3 64m 2in 37, G4 38.36 39. 16n-32 40. 72Rem* 41 10 me 127 =4,R= 3 43.12" 44 S~ 49.15 1 6, i 46. 55 47.0 LA a A LE aC Ee A i ¢ 9B 2D 40. © 2 8B 10. B a: 10. D 2B 10. ¢ 3D WA 3c fds at 3.28 wc 3D uu B ar 2 Dd aa RE ac RE ac RE aE 13. A & D 13. £ aE 13. D 8 1B. A 6 C 4B & A 14. c 6 8B 14. c 6 MA LE io. B TA 1. E 7D 13 A i 1b. E & D 16. B 8D 16. B 8 E 16. A 8 16. D Aruwer to Exerier ‘altitude. a lime segment between a vertex of a polygon and the Line containing a side of the polygon, which 1s, perpendicular to this bne. ‘augle bisector a line or line segment that divides an angle into two equal parts. apotliem. the distance between the center of a regular polygon and any one of its sides. ‘area part of a cirele which is bounded by two points on the excle. area. The area of a surface 1s the total number of ‘non-overlapping square units and part units that cover it ‘base. a side ofa polygon from which we draw an altitude. c center of @ regula’ polygon: The centers of the mseribed and cxcumscnibed circles of a regular polygon comeide at the same point. This pomt 1s called the center of the polygon. central eugle am angle whose vertex lies at the center of a circle. centroid: the intersection point of the medians of a tnange. chord a line segment which comieets two points on a carcle. ‘orcle: the set of all points which are equidistant from. a fixed point. ‘chreular region: the union of a circle and ats interior region, ciyewmrad ius: the radius of a cirumsenibed circle. chroumscribe: Ifa figure P : circumscribes a polygon P, all the vertices of P he on F. ‘chrcwmsertbed evycle- a circle drawn around a polygon ‘which passes through all the vertices of the polygon. concentric eircles two or more carcles which have different radii but the same center, diagonal a ime segment which joins two non-adjacent vertices of a polygon. cameter of @ circle. a chord of a circle that passes through the center of the circle. eouianpilar polyoon: a polygon whose angles all have the same measure equilateral polygon a polvgon whose sides are all the same length equiloteral triangle a triangle with thee congruent sides and three 60” angles. height the length of an alutude of a polygon. hexagon a polygon with six sides. incenter’ the center of the inscribed circle of a polygon. The incenter of a tangle is the mrersection, pomt of Hs angle bisectors. invadius the radius of the mseribed circle of a polygon. insertbed gugle: an angle ABC formed by three pomts ‘A, Band C on a cnele inseribed circle’ a etrele drawn mside a polygon which AU1s tangent (0 every side of the polygon. inseribed polygon. if a polygon P is amseribed m a figure F, all the vernices of P he on F. isosceles fraperoid a trapezoid whose two non-parallel ‘sides haye equal length. isosceles criaugle a triangle with at least two congruent sides. K ite a quadrilateral that has two pairs of adjacent congruent sides L legs (of a right triangle): the two sides adjacent to the right angle in a right triangle legs (Of @ Waperotd) the two non-parallel sides of 2 trapezoid, median (of tapecoid) the Ime segment that connects the midpoints of the lees of a trapezoid. ‘median (of & tiongle) a segment whose endpomts are the vertex of the tangle and the midpoint of the side opposite the vertex. Every triangle has three medians, ° ‘octagon, a polygon with eight sides, P pavallelogram a quadrilateral whose opposite sides are congruent and parallel to each other. pentagon a polygon with five sides. perimeter the total Length of the Lne which encloses a region mn a plane. The perimeter of a polygon 1s the sum of the lengths of 1ls sides, The permeter of a eurcle is its exrcumtference. polygonal woion the union of a polygon and ms interior region. 2 ‘quadrilateral. a polygon with four sides. quarter eile: a sector ofa circle whose central angle 15 00°. A quarter circle is one-fourth of a circle rodion a unit of angle measure. A whole angle measures 2n radians. rodins: the distance between the center of a circle and. any point on the circle, rectangle. a quadnlateral whose opposite sides are parallel to each other and of equal length, and whose interior angles are all nght angles. vegular golygon- a polygon which is both equilateral and equiangular, rhombus. a parallelogram whose sides all haye equal Jength. right tapezoid. a trapezoid with exactly two right anges. pight triangle. a tangle with one right angle, ving. the area between two concentnec eitcles. s sector” a region bounded by two radii of a elrcle and the arc between the endpoints of these radii segment a part ofa circular region which is bounded by a chord and the are between the endpoints of Us chord. semicircle’ a sector of a circle which has a central ange of 180°, A semicircle is half of a etree square a parallelogram which has four congruent sides and four night angles. A square is both a shombus and a rectangle. square nit the interior region of a square with side Jength one unt. i tangent tinea ne which intersects a circle at only one pom. vvaperotd: a quadnilateral with two parallel sides. triaugalar region: the union of a triangle and its interior region, Vv vertex af @ polygon the common endpoint of any two adjacent sides of a polygon.

You might also like